Internal medicine Flashcards

1
Q
A
How well did you know this?
1
Not at all
2
3
4
5
Perfectly
2
Q

20.1 Cardiovascular effects of hyperthyroidism include

a) decreased diastolic relaxation
b) decreased SVR
c) decreased PVR
d) increased diastolic BP

A

Decreased SVR
- increased CO, increased SBP and decreased DBP with widened PP

Up to Date
Cardiovascular - Patients with hyperthyroidism have an increase in cardiac output, due both to increased peripheral oxygen needs and increased cardiac contractility. Heart rate is increased, pulse pressure is widened, and peripheral vascular resistance is decreased

How well did you know this?
1
Not at all
2
3
4
5
Perfectly
3
Q

23.1 During standard diagnostic nocturnal polysomnography for investigation of obstructive sleep apnoea, apnoea is defined as cessation of airflow for

A. 10 sec
B. 20 sec
C. 30 sec
D. 10 sec with 3% desat
E. 20 sec with 3 % desat

A

A

Apnea is defined as the cessation of airflow for ten or more seconds.

Hypopnea is defined as a recognizable, transient reduction, but not a complete cessation of, breathing for ten or more seconds.

Hypopnea requires a 4% fall in SpO2

https://www.ncbi.nlm.nih.gov/books/NBK441909/#:~:text=Obstructive%20Sleep%20Apnea%20(OSA)%2C,for%20ten%20or%20more%20seconds.

How well did you know this?
1
Not at all
2
3
4
5
Perfectly
4
Q

20.1 A 64-year-old man presenting for elective surgery is on thyroxine 100 mcg daily. His thyroid function tests are: (Thyroid function tests shown). These results are most consistent with

TFTs thryoxine TSH < .05 T4 and T3 completely normal

a) Hypophysectomy
b) Subclinical Hyperthyoirdism
c) Sick euthyroid
d) Toxic Multinodular goitre

A

b) Subclinical Hyperthyoirdism

Subclinical hyperthyroidism: low TSH, normal T3 + T4
Clinical hyperthyroidism: low TSH, high T3, high/normal T4

Subclinical hypothyroidism: high TSH, normal T3 + T4
Clinical hypothyroidism: high TSH, low/normal T3, i T4

Amiodarone: high/normal TSH, low T3 (2o to inhibition of pituitary T4 to T3 conversion)

Sick euthyroid: low TSH, low T3

Hypophysectomy (central hypothyroidism): low/normal TSH/T3/T4

Compliant on thyroxine: normal TSH, high/normal T3, low T4
Non-compliant w thyroxine (pt taking several tabs prior to Dr’s appointment): high TSH, normal T4

How well did you know this?
1
Not at all
2
3
4
5
Perfectly
5
Q

21.2 Of the following drugs, the least likely to cause pulmonary vasodilation when used at low
doses in patients with chronic pulmonary hypertension is

a) Dopamine
b) Dobutamine
c) Vasopressin
d) Milrinone

A

dopamine

  • least likely to cause pulmonary vasodilation (all the others do to my knowledge)
  • From UP TO DATE:
    > At low doses of 1 to 3 mcg/kg per min, dopamine acts primarily on dopamine-1 receptors to dilate the renal and mesenteric artery beds
    > At 3 to 10 mcg/kg per min (and perhaps also at lower doses), dopamine also stimulates beta-1 adrenergic receptors and increases cardiac output, predominantly by increasing stroke volume with variable effects on heart rate.
    > At medium-to-high doses, dopamine also stimulates alpha-adrenergic receptors, although a small study suggested that renal arterial vasodilation and improvement in cardiac output may persist as the dopamine dose is titrated up to 10 mcg/kg per min
    *clinically, the haemodynamic effects of dopamine demonstrate individual variability

Dobutamine (inodilator):
- selective β1-agonist that increases cardiac contractility and reduces pulmonary vascular and systemic vascular resistances

Vasopressin:
- vasopressin may have pulmonary vasodilatory effects in addition to a systemic vasoconstrictive effect

Milrinone (inodilator):
- the phosphodiesterase-3 inhibitors, milrinone and enxoimone, have positive inotropic effects combined with the capacity to reduce RV afterload (‘inodilators’) without significant chronotropic effect, but they can be associated with significant systemic hypotension

How well did you know this?
1
Not at all
2
3
4
5
Perfectly
6
Q

23.1 In patients with primary adrenal insufficiency, a markedly elevated renin is most likely due to

A Insufficient corticosteroid replacement
B Insufficient fludrocortisone replacement
C Excessive corticosteroid replacement
D Excessive fludrocortisone replacement

A

b. Insufficient fludrocortisone replacement

In Primary Adrenal Insufficency, cortisol deficiency results in decreased feedback to the HPA axis, leading to increased secretion of ACTH to stimulate the adrenal cortex. Simultaneously, MCs deficiency causes increased release of renin by the juxtaglomerular apparatus of the kidneys.

How well did you know this?
1
Not at all
2
3
4
5
Perfectly
7
Q

22.1 A 30-year-old parturient presents in labour. She has a history of Addison’s disease from autoimmune adrenalitis and has been taking prednisolone 6 mg daily for ten years. On presentation the patient is given hydrocortisone 100 mg intravenously. The most appropriate steroid replacement regimen the patient should receive during labour is

a. 25mg TDS hydrocortisone
b. 8mg/hr hydrocortisone
c. 6mg PO prednisone

A

8mg/hr

Guidelines for mx of glucocorticoids during the perioperative period for patients with adrenal insufficiency

https://associationofanaesthetists-publications.onlinelibrary.wiley.com/doi/10.1111/anae.14963

How well did you know this?
1
Not at all
2
3
4
5
Perfectly
8
Q

21.1 A 45-year-old man has the following results on his blood biochemistry testing (Liver function tests shown). The most likely diagnosis is

a. Cholecystitis
b. Metastatic liver disease
c. Hepatitis C
d. Chronic liver disease
e. Paracetamol toxicity

A

a. Cholecystitis

Example and explanation taken from RACGP:
The raised AlP relative to Alt suggests cholestasis and the high GGt confirms liver origin. The mild hyperbilirubinaemia confirms the clinical impression of jaundice. Biliary disease is highly likely with gallstones the most likely differential diagnosis. however, this clinical picture may also occur in drug reactions or infiltrative conditions. After a careful history, abdominal ultrasound is the most appropriate next investigation.

How well did you know this?
1
Not at all
2
3
4
5
Perfectly
9
Q

22.1 Complications of severe anorexia nervosa (body weight < 40% ideal) include all of the following EXCEPT

a. HypoK
b. Cl abnormality
c. Delayed gastric emptying
d. Hypercalcaemia
e. Cardiomyopathy

A

Hypercalcaemia

How well did you know this?
1
Not at all
2
3
4
5
Perfectly
10
Q

22.1 A 45-year-old man presents on the day of surgery for an elective inguinal hernia repair. He is well
but is noted to be mildly jaundiced. He takes simvastatin for hyperlipidaemia and has no other medical history. He consumes about three standard drinks of alcohol per day and does not smoke. He briefly experimented with illicit drugs more than ten years ago. His laboratory results show: (supplied) The most likely diagnosis is

Normal electrolytes
ALP 85 N
ALT 31 N
AST 31 N
GGT 15 N
Urea 10 [4-9]
Creatinine 103 N
Total protein 74 N
Albumin 40 N
BSL 4.2 N
Bilirubin 29 [0-20]
Conjugated 5
Unconjugated 24

A. Fatty liver
B. Hepatitis
C. Cholestasis
D. Gilbert syndrome
E. Drug induced

A

Gilberts

Gilbert’s syndrome is a benign genetic condition that commonly presents as incidental
hyperbilirubinaemia or painless jaundice.

It is relatively common with a population frequency of approximately 2–10%.

Gilbert’s syndrome is caused by defective bilirubin clearance by the hepatic conjugating enzyme UDP-glucuronosyltransferase

https://www.rcpa.edu.au/getattachment/8b9a8acf-f7f5-4088-951c-3f65f0c2f8fe/Interpreting-liver-function-tests.aspx

How well did you know this?
1
Not at all
2
3
4
5
Perfectly
11
Q

21.2 The condition in which volatile anaesthesia is least appropriate is

a) Multiple sclerosis
b) Myasthenia gravis
c) Lambert-Eaton syndrome
d) Guillain-Barre syndrome
e) Muscular dystrophy

A

e) Muscular dystrophy
- rhabdomyolysis risk if given to patients with Duchenne or Becker’s muscular dystrophy
- volatiles safe in all above, and also safe in patient’s with myotonic dystrophy

Malignant hyperthermia
- high mortality uncoupling regulation of RyR1 to SR
Duschenne muscular dystrophy
- fatal rhabdo (hyperkalaemia)

How well did you know this?
1
Not at all
2
3
4
5
Perfectly
12
Q

21.2 An electrocardiogram (ECG) abnormality which is NOT usually associated with severe anorexia nervosa is

a) Sinus tachycardia
b) Wandering atrial pacemakers
c) ST depression
d) T wave inversion
e) Prolonged QT

A

a) Sinus tachycardia

BJA: Anorexia nervosa: perioperative implications
https://academic.oup.com/bjaed/article/9/2/61/299563

Cardiovascular

Typically anorexic patients are hypotensive and bradycardic. These physiological markers may be used as indications for hospitalization.

Bradycardia reflects the decrease in basal metabolic rate that arises as an adaptive response to starvation. Although patients are usually in sinus rhythm, electrocardiographic abnormalities are common and may be found in >80% of strict dieters. These include: atrioventricular block, ST depression, T wave inversion, and QT prolongation.

QT prolongation may be caused by hypocalcaemia, hypomagnesaemia, drugs, or directly by starvation itself. Electrolyte disturbances have a significant causal role in ECG abnormalities.

Other factors, for example, atypical antipsychotics, may also contribute.

Associated arrhythmias include: sinus arrest, wandering atrial pacemakers, nodal escape beats, supraventricular tachycardia, and ventricular tachycardia.

The reported incidence of arrhythmias under anaesthesia is 16–62%.

With respect to myocardial contractility, left ventricular function has been demonstrated to be impaired in a proportion of patients. Echocardiographic studies have also demonstrated a higher incidence of mitral valve prolapse in anorexic patients. The reasons for this are not entirely clear. It is postulated that the loss of left ventricular volume and mass leads to abnormal mitral valve motion.

In addition to starvation-induced myocardial impairment, the myocardium may be specifically damaged by pharmacological agents.

For example, emetogenic ipecac syrup is directly cardiomyotoxic and produces inflammatory changes and myocardial fibre degeneration when used long term.

Rarely, antipsychotic drugs, for example, olanzapine, may cause cardiomyopathy.

Myocardial impairment can be caused by hypophosphataemia which also reduces the threshold for arrhythmias.

Compromised myocardial function requires judicious use of fluids perioperatively as there is an increased risk of congestive cardiac failure. Echocardiography along with invasive perioperative monitoring (central venous catheter) should be considered to prevent fluid overload.

How well did you know this?
1
Not at all
2
3
4
5
Perfectly
13
Q

During a routine preoperative examination of a patient’s heart, you note exaggerated splitting of the second heart sound with inspiration. This is characteristically heard in

A. Aortic Reguritation
B. HOCM
C. Left bundle branch block
D. Mitral Stenosis
E. Pulmonary Stenosis

A

E. Pulmonary Stenosis

DERANGED PHYSIOLOGY:

Splitting of the first heart sound
Right bundle branch block can produce a split first heart sound - because the contraction of the right ventricle is delayed- the conduction occurs via the left ventricle rather than the bundle of His- and thefore the closure of the tricuspid valve occurs after a substantial delay.
Atrial septal defect can result in a fixed split of the first heart sound

Splitting of the second heart sound

It is normal for this sound to be split. The high pressure in the systemic circulation slams the aortic valve shut rather abruptly, almost angrily. In contrast, low pressure of the pulmonary circulation tends to close the pulmonary valve gently, and therefore the pulmonary component of the second heart sound (P2) is usually delayed by about 20-30 milliseconds.

It is also normal for increased right ventricular filling to cause a widening of the split. The more blood in the RV, the longer it takes to eject, and therefore the greater the delay until pulmonary valve closure.

n the spontaneously breathing patient, the delay is greatest during inspiration. Naturally, in the patient ventilated with positive pressure the delay is greatest during expiration (positive pressure being a barrier to diastolic filling).

Increased normal splitting of S2

Anything that delays the end of right ventricular systole can cause this sort of picture.

Right bundle branch block - the delay in conduction via the left ventricle causes a delay in right ventricular contraction, and therefore a delay in pulmonary valve closure. The S1 will also be split.
Ventricular septal defect - because the right ventricle receives a large volume load directly from the left ventricle, and therefore takes longer to complete its systolic contraction.

Pulmonary valve stenosis - because the right ventricle takes longer to empty though a narrowed valve

Mitral regurgitation- not because right ventricular contraction is delayed, but because left ventricular contraction is shortened (as the LV empties in both the aortic and the atrial directuion, systole is over very quickly).

Fixed splitting of S2

Atrial septal defect - the atria, joined by a gaping hole in their seput, act as one atrium. The result is a reasonably equal distribution in volume betweent the right and left atrium. This way, both sides of the circulation share the same diastolic filling pressure. Dragging more volume into the right atrium with respiratory activity will not cause an inequality of ventricular filling (between the right and left ventricles) because the venous return will be “shared”.

Reversed splitting of S2

In this situation, P2 occurs before A2, and splitting widens during expiration (or inspiration in the mechanically ventilated patient). This only happens if the conduction to the left ventricle is delayed, or if the left ventricle is massively volume overload (and the right ventricle is not).
Left bundle branch block - the left ventricle depolarises after the right ventricle, and A2 is delayed
Aortic stenosis - the left ventricle empties slowly though a narrow valve
Large patent ductus arteriosus - the left ventricle receives a backflow of blood from the aorta, which causes it to become volume-overloaded

How well did you know this?
1
Not at all
2
3
4
5
Perfectly
14
Q

21.1 A patient with C6 tetraplegia is undergoing removal of bladder stones under general anaesthesia. The blood pressure rises to 166/88 mmHg. The appropriate response is to

a. Clonidine
b. Hydralazine
c. Decompress the bladder
d. Fentanyl
e. Deepen your anaesthetic

A

decompress the bladder

Autonomic Dysreflexia:
- medical emergency characterised by severe hypertension,
- brought on by stimulation below the level of the lesion

Factors affecting the development of ADR:
1. Level of spinal injury
2. Duration of injury
3. Whether injury is complete or incomplete

Pathology:
Stimuli arise from caudal roots below the level of the lesion leading to uncontrolled sympathetic activation below the level of the lesion
○ 80% being due to bladder distension
○ Other triggers include
§ bowel distension
§ acute abdo pathology
§ activation of pain fibres
§ sexual activity
§ uterine contractions

How well did you know this?
1
Not at all
2
3
4
5
Perfectly
15
Q

22.2 A 50-year-old man has the following pulmonary function test result: (provided). The most consistent diagnosis is
FEV1 68%, FVC 68%, DLCO 91%

a. Pulmonary hypertension
b. pulmonary fibrosis
c. myasthenia gravis
d. sarcoidosis

A

c. myasthenia gravis

How well did you know this?
1
Not at all
2
3
4
5
Perfectly
16
Q

21.2 A 25-year-old male has continued post operative bleeding after an extraction of an impacted third molar tooth under a general anaesthetic. The patient mentions that his father bruises quite easily. His coagulation screen reveals: (Coagulation tests provided). The most likely diagnosis is

His coagulation screen reveals: Prolonged APTT, Normal PT.

a) Factor V Leiden
b) Haemophilia A
c) Haemophilia B
d) Von willebrand disease

A

d) Von willebrand disease
- autosomal dominant inheritance
- may have normal or prolonged APTT, PT is normal

*Haem A: X-linked recessive disorder; would expect prolonged aPTT, and normal PT
*Haem B: X-linked recessive disorder; would expect normal aPTT and normal PT

Up to date:
Inheritance patterns — Most cases of VWD are transmitted as an autosomal dominant trait; this includes types 1 and 2B, and most types 2A and 2M.

Baseline hemostasis assessment —
Most patients will have a complete blood count (CBC) with platelet count and coagulation studies during the initial evaluation for excessive bleeding or bruising.
●Individuals with VWD generally have a normal CBC and a normal platelet count, with the exception of those with type 2B VWD, most of whom will have mild thrombocytopenia (eg, platelet count 100,000 to 140,000/microL).
●Individuals with VWD may have a normal or prolonged activated partial thromboplastin time (aPTT), depending on the degree of reduction of the factor VIII level. The prothrombin time (PT) is normal in VWD.

Up to date:
●Hemophilia A – Inherited deficiency of factor VIII (factor 8 [F8]); an X-linked recessive disorder.
●Hemophilia B – Inherited deficiency of factor IX (factor 9 [F9]); also called Christmas disease; an X-linked recessive disorder.

Laboratory findings —
Hemophilia is characterized by a prolonged activated partial thromboplastin time (aPTT).
However, the aPTT may be normal in individuals with milder factor deficiencies (eg, factor activity level >15 percent), especially in hemophilia B (factor IX deficiency), where even individuals with moderate disease may have a normal aPTT.
In some individuals with hemophilia A, factor VIII levels may increase with stress, leading to a normalization of the aPTT or mis-categorization of factor levels and disease severity.
In patients with hemophilia, the aPTT corrects in mixing studies, unless an inhibitor is present, which only applies to individuals who have received factor infusions or who have an autoantibody such as a lupus anticoagulant or an acquired factor inhibitor.
Mixing studies that do not show correction of a prolonged aPTT suggest an alternative diagnosis such as an acquired factor inhibitor.
The platelet count and prothrombin time (PT) are normal in hemophilia.
Thrombocytopenia and/or prolonged PT suggest another diagnosis instead of (or in addition to) hemophilia.
Measurement of the factor activity level (factor VIII in hemophilia A; factor IX in hemophilia B) shows a reduced level compared with controls (generally <40 percent).
One exception is an individual with mild hemophilia A who undergoes testing when stressed or pregnant and has a falsely elevated factor level. If this is suspected, factor activity testing should be repeated under conditions of low stress.
The plasma von Willebrand factor antigen (VWF:Ag) is normal in hemophilia.
If VWF:Ag is reduced, this suggests the possibility of von Willebrand disease (VWD) rather than (or in addition to) hemophilia.
Urinalysis is not done routinely, but if performed it may sometimes (but not always) show microscopic or macroscopic hematuria.

How well did you know this?
1
Not at all
2
3
4
5
Perfectly
17
Q

23.1 In a patient with glucose-6-phosphate dehydrogenase deficiency (G6PD), the
intravenous agent that should be avoided is

a. Methylene blue
b. Indocyanine green (ICG)
c. Iodine
d. Dextrose

A

a) methylene blue

Drugs to avoid:

Antibiotics
Sulphonamides (check with your doctor)
Co-trimoxazole (Bactrim, Septrin)
Dapsone
Chloramphenicol
Nitrofurantoin
Nalidixic acid

Antimalarials
Chloroquine
Hydroxychloroquine
Primaquine
Quinine
Mepacrine

Chemicals
Moth balls (naphthalene)
Methylene blue

Foods
Fava beans (also called broad beans)

Other drugs
Sulphasalazine
Methyldopa
Large doses of vitamin C
Hydralazine
Procainamide
Quinidine
Some anti-cancer drugs

How well did you know this?
1
Not at all
2
3
4
5
Perfectly
18
Q

20.1 The neurosurgical registrar has telephoned about a patient with a spinal cord tumour who is on the list for tomorrow. The registrar tells you the patient has Brown-Séquard syndrome (hemisection of the spinal cord). On clinical examination, below the level of the lesion, you would expect to find all EXCEPT ipsilateral

A. Hyperreflexia
B. Loss of tactile stimulation
C. Paralysis
D. Loss of pain/temperature
E. Loss of vibration/proprioception

A

D. Loss of pain/temperature

Brown-Sequard syndrome:
- Also known as Lateral hemi-section syndrome
- Causes
○ Common
§ Knife or bullet injuries
§ Demyelination
○ Rare
§ Spinal cord tumours
§ Disc herniation
§ Infarction
§ infection
Ipsilateral:
- Motor weakness
- Loss of vibration sensation
- Loss of proprioception sensation
Contralateral:
- Loss of pain sensation
- Loss of temperature

Segmental Syndrome:
- Pathologies that affect all functions of the spinal cord at one or more levels
- Total cord transection:
○ Cessation of function in all ascending and descending spinal cord pathways
○ Loss of all types of sensation below the level of the lesion
○ Loss of movement below the level of the lesion
- Acute transection:
○ Spinal shock
○ Flaccid paralysis
○ Urinary retention
○ Diminished tendon reflexes
○ This is usually temporary followed by:
§ Increased tone
§ Spasticity
§ Hyperrelfexia
§ supervene days or weeks after the event
- Transverse injuries above C3 involve sensation of respiration and are often fatal if acute
- Lesions above L2 will cause impotence and spastic paralysis of bladder
- Causes:
○ Myelopathies
§ Traumatic injury
§ Spinal cord haemorrhage
○ Epidural or intramedullary abscesses or tumours and transverse myelitis may have a more subacute presentation

Dorsal (posterior) cord syndrome:
- Bilateral involvement of:
○ Dorsal Columns
○ Corticospinal tracts
○ Descending central autonomic tracts to bladder control centres in the sacral cord
- Symptoms/signs:
○ Gait Ataxia (DC)
○ Paraesthesias (DC)
○ Weakness (CST)
§ Acute
□ Muscle flaccidity
□ Hyporeflexia
§ Chronic
□ Muscle hypertonia
□ Hyperreflexia
○ Extensor plantar response
○ Urinary incontinence (Auto)
- Causes:
○ MS
○ Tabes dorsalis
○ Friedrich ataxia
○ Sub-acute combined degeneration
○ Vascular malformations
○ Epidural and intradural extrameduallry tumours
○ Atlantoaxial subluxation

Ventral (anterior) cord syndrome
- Involves cords in the anterior 2/3rds of the spinal cord
○ Corticospinal tract
○ Spinothalamic tract
○ Descending autonomic tracts to the sacral centers for bladder control
- Signs/symptoms
○ Weakness (CST)
○ Reflex changes (CST)
○ B/L temp and pain sensation (Spino)
○ Tactile and vibratory sense are normal
○ Urinary incontinence (Auto)
- Causes:
○ Spinal cord infarction
○ Intervertebral disc herniation
○ Radiation myelopathy

How well did you know this?
1
Not at all
2
3
4
5
Perfectly
19
Q

23.1 A patient with long-term severe anorexia nervosa is commenced on a normal diet. Three days later she develops cardiac failure and exhibits a decreased level of consciousness. The most important parameter to assay and normalise is the plasma

a. Phosphate
b. Potassium
c. Magnesium
d. Sodium
e. Calcium

A

a) Phosphate

hypophosphate: Clinical symptoms range from muscle weakness and paraesthesia to severe cardiac failure, seizures and diaphragmatic paralysis

Refeeding malnourished patients with anorexia nervosa can be associated with hypophosphatemia, cardiac arrhythmia and delirium. Phosphorus repletion should be started early with and serum levels maintained above 3 mg/dL

weakness and fatigue, in the context of a recent history of starting a regular diet while in a state of chronic malnutrition, are concerning for refeeding syndrome, which typically occurs 2 to 5 days after beginning nutritional repletion. Depleted phosphate stores due to prolonged starvation, hypocalcemia, and hypokalemia can lead to impaired muscle contractility and subsequently weakness, myalgia, and tetany.

https://www.ncbi.nlm.nih.gov/pmc/articles/PMC4168120/

How well did you know this?
1
Not at all
2
3
4
5
Perfectly
20
Q

22.1 A 26-year-old patient presents with exertional syncope. The most likely diagnosis is

a. HOCM
b. Long QT syndrome
c. CCF
d. IHD

A

HOCM: pathopneumonic

A person who has syncope during exertion is more likely to have an obstruction to blood flow (aortic stenosis or hypertrophic cardiomyopathy) or ventricular tachycardia as a cause. On the other hand, syncope after completion of exercise is more likely of reflex origin, such as the common faint.

https://www.uptodate.com/contents/syncope-fainting-beyond-the-basics#:~:text=A%20person%20who%20has%20syncope,such%20as%20the%20common%20faint

How well did you know this?
1
Not at all
2
3
4
5
Perfectly
21
Q

22.1 Created by the Global Initiative for Chronic Obstructive Lung Disease, the alphabetical GOLD groups A to D are tools for the assessment of chronic obstructive pulmonary disease. These classes are based on

a. Symptoms and exacerbations
b. FEV1
c. FEV1 and exacerbations
d. FEV1/FVC and exacerbations
e. FEV1 and symptoms

A

Sx and exacerbations

FEV1

GOLD ABE assessment tool

How well did you know this?
1
Not at all
2
3
4
5
Perfectly
22
Q

A 50-year-old man has the following pulmonary function test result. The most consistent diagnosis is

FEV1 98% predicted
FVC 98% predicted
DLCO 48% predicted

a) Asthma
b) Obesity
c) Sarcoidosis
d) Pulmonary hypertension

A

d) Pulmonary hypertension
Normal spirometry + low DLCO

Asthma: obstructive pattern and normal DLCO
Obesity: restrictive pattern and normal DLCO
Sarcoid: restrictive pattern and low DLCO

How well did you know this?
1
Not at all
2
3
4
5
Perfectly
23
Q

21.2 Cardiovascular effects of hyperthyroidism include

a) Increased DBP
b) Narrow pulse pressure
c) Reduced diastolic relaxation
d) Decreased CO
e) Decreased SVR

A

e) Decreased SVR
- increased CO, increased SBP and decreased DBP with widened PP

UP TO DATE: Cardiovascular effects of hyperthyroidism:

  • Thyroid hormone has important effects on cardiac muscle, the peripheral circulation, and the sympathetic nervous system that alter cardiovascular hemodynamics in a predictable way in patients with hyperthyroidism.
  • The main changes are :
    ●Increases in heart rate, cardiac contractility, systolic and mean pulmonary artery pressure, cardiac output, diastolic relaxation, and myocardial oxygen consumption
    ●Reductions in systemic vascular resistance and diastolic pressure
How well did you know this?
1
Not at all
2
3
4
5
Perfectly
24
Q

21.2 The risk of postoperative respiratory failure in myasthenia gravis is increased by the
administration of

a) Teicoplanin
b) Flucloxacillin
c) Cephazolin
d) Gentamicin
e) Vancomycin

A

d) Gentamicin

Drugs in the anaesthetic trolley that may unmask or worsen MG:
- NMBs
- gentamicin
- beta blockers (metoprolol)
- magnesium

Anaesthetic drugs to be cautious with:
- dexamethasone
- antipsychotics
- anticonvulsants
- antibiotics (vancomycin, metronidazole)

How well did you know this?
1
Not at all
2
3
4
5
Perfectly
25
Q

22.2 Of the following, the congenital condition LEAST commonly associated with obstructive sleep apnoea in children is

A

Hypoplastic mandible (micrognathia) – difficult intubation
§ Pierre Robin sequence
§ Treacher Collins
§ Hemifacial microsomia (Goldenhar syndrome)

Midface hypoplasia – difficult bag-mask ventilation
§ Apert syndrome
§ Crouzon syndrome
§ Pfeiffer syndrome
§ Saethre-Chotzen syndrome

Macroglossia – difficult bag-mask ventilation AND difficult intubation
§ Hurler’s/Hunter’s syndrome (mucopolysaccharidoses)
§ Beckwith-Wiedemann syndrome
§ Down’s syndrome

https://www.frca.co.uk/Documents/250%20The%20Difficult%20Paediatric%20Ai

Mucopolysaccharidoses, Down syndrome, muscular dystrophies, and other neurologic disorders have been associated with obstructive sleep apnea

Prevalence of OSAS.
Genetic Disorder Prevalence of OSAS
Neuromuscular diseases 69.2%
Prader–Willi syndrome 94.7%
Arnold–Chiari syndrome 80%
Achondroplasia 100%
Crouzon syndrome 100%
https://www.ncbi.nlm.nih.gov/pmc/articles/PMC8156845/

https://www.frca.co.uk/Documents/250%20The%20Difficult%20Paediatric%20Ai

How well did you know this?
1
Not at all
2
3
4
5
Perfectly
26
Q

20.1 A 55-year-old lady scheduled for a transphenoidal hypophysectomy undergoes an oral glucose tolerance test with the following results:

GH normal <10
Time 0, BSL 5.5, GH 30, IGF-1 790 (elevated)
Time 30, BSL 7.6, GH 24
Time 60, BSL 7.2, GH 28
Time 90, BSL 6.5, GH 26
Time 120, BSL 5.8, GH 29

These results are most consistent with a diagnosis of

A. Prolactinoma
B. Acromegaly
C. Cushing’s
D. MEN 2
E. Normal

A

Acromegaly

IGF-2 is consistently elevated

GH should be suppressed by glucose load in healthy
pt.

The continued elevation of GH despite glucose is
suggestive of acromegaly

How well did you know this?
1
Not at all
2
3
4
5
Perfectly
27
Q

23.1 In children, severe sleep apnoea is suggested by an apnoea-hypopnoea index
greater than

a. 10
b. 15
c. 20
d. 30
e. 40

A

a) 10

How well did you know this?
1
Not at all
2
3
4
5
Perfectly
28
Q

21.2, 20.1 The muscle or muscle group with the greatest resistance to the action of non-depolarising neuromuscular blocking agents is the

a) Adductor pollicis
b) Diaphragm
c) Orbicularis oculi
d) Pharyngeal

A

b) Diaphragm

How well did you know this?
1
Not at all
2
3
4
5
Perfectly
29
Q

20.1 RFTS: Normal ratio, low FVC, low FEV1, Normal DLCO:
a) Sarcoid
b) Myasthenia Gravis
c) Asthma
d) Emphysema

A

b) Myasthenia Gravis

How well did you know this?
1
Not at all
2
3
4
5
Perfectly
30
Q

21.2 In a patient with anaemia of chronic disease, of the following the most likely to be elevated is

a. MCV
b. transferrin saturation
c. Increased soluble Transferrin Receptor
d. Ferritin
e. Total iron binding capacity

A

d. Ferritin

ANZCA blue book:

ACD caused primarily by inflammation

Mechanism:
1. Iron
- Inflammation reduces Iron availabilty as a protective mechanism whereby Iron is sequestered and stored in macrophages to limit availability to microbial pathogens
- Hepcidin expression is increased, this prevents the release of Iron by reticuloendothelial system resulting in “functional iron deficiency” with reduced tissue availability of iron, despite apparently normal total body iron stores. (hence increased Ferritin)

  1. Response to erythropoietin
    - mechanism not clear suspect blunting of response to erythropoietin
  2. Therapeutic agents
    chemotherapies that impair bone marrow response to erythropoiesis
    65% of patients with lung and gynae cancer treated with platinum based drug develop anaemia

RCPA advice on interpretation of Soluble Transferrin Receptor:

Soluble transferrin receptor levels in plasma are elevated if there is increased iron demand due to Iron deficiency, increased erythropoiesis (eg, Haemolysis) or dyserythropoiesis (eg, Megaloblastic anaemia), regardless of other, coexistent states.

Thus, it can be used to demonstrate iron deficiency in patients who also have an acute phase response and it can distinguish Iron deficiency from the Anaemia of chronic disease.

Patients with an acute phase response have reduced plasma iron and transferrin with elevation of Ferritin, making these usual indicators unreliable.

How well did you know this?
1
Not at all
2
3
4
5
Perfectly
31
Q

22.2 Suxamethonium may be safely given to patients with (list of neuromuscular diseases given)

a. Becker muscular dystrophy
b. Myaesthenia gravis (new option)
c. Guillain Barre
d. Hypokalaemic periodic paralysis (new option)
e. Duchenne muscular dystrophy

A

b. Myaesthenia gravis

ED95 is 0.8mg/kg in a MG patient

How well did you know this?
1
Not at all
2
3
4
5
Perfectly
32
Q

22.2 You will anaesthetise a 39-year-old woman for a laparoscopic cholecystectomy. She has a history of mastocytosis and has never had an anaesthetic in the past. A drug which you should avoid is
a. fentanyl
b. morphine
c. remifentanil
d. tramadol

A

B Morphine

Histamine-releasing

How well did you know this?
1
Not at all
2
3
4
5
Perfectly
33
Q

23.1 Expected features of Guillain-Barré syndrome include

A. Descending paralysis
B. Flaccid paralysis
C. Unilateral leg weakness

A

b) flaccid paralysis

Guillain–Barré syndrome (GBS) is an inflammatory disease of the PNS and is the most common cause of acute flaccid paralysis

How well did you know this?
1
Not at all
2
3
4
5
Perfectly
34
Q

21.1 Perioperative overheating is most likely to cause worsening of symptoms of

a) Duchenne Muscular dystrophy
b) Myasthenia gravis
c) Multiple sclerosis
d) Myotonica dystrophia
e) Eaton Lambert syndrome

A

multiple sclerosis.

https://academic.oup.com/bjaed/article/11/4/119/266998
Anaesthetic considerations.
- Local anaesthetics may exacerbate symptoms due to the increased sensitivity of demyelinated axons to local anaesthetic toxicity.
- Non-depolarizing neuromuscular blocking agents may be used in normal doses.
Caution should be exercised when using depolarizing neuromuscular blocking agents if the patient is debilitated.
- Temperature maintenance is important as symptoms can deteriorate with an increase in temperature, as demyelinated axons are also more sensitive to heat.

How well did you know this?
1
Not at all
2
3
4
5
Perfectly
35
Q

22.2 The 2012 Berlin definition of the acute respiratory distress syndrome (ARDS) defines moderate disease as one with a PaO2 / FiO2 ratio (in mmHg) of

a) 50-100
b) 100-200
c) 200-300
d) 300-400

A

a) 100-200

2012 BERLIN DEFINITION OF ARDS

ARDS is an acute diffuse, inflammatory lung injury, leading to increased pulmonary vascular permeability, increased lung weight, and loss of aerated lung tissue…[with] hypoxemia and bilateral radiographic opacities, associated with increased venous admixture, increased physiological dead space and decreased lung compliance.

Key components
- acute, meaning onset over 1 week or less
- bilateral opacities consistent with pulmonary edema must be present and may be detected on CT or chest radiograph
- PF ratio <300mmHg with a minimum of 5 cmH20 PEEP (or CPAP)
- “must not be fully explained by cardiac failure or fluid overload,” in the physician’s best estimation using available information — an “objective assessment“ (e.g. echocardiogram) should be performed in most cases if there is no clear cause such as trauma or sepsis.

Severity
- ARDS is categorized as being mild, moderate, or severe:

How well did you know this?
1
Not at all
2
3
4
5
Perfectly
36
Q

20.1 A 22-year-old patient is scheduled for resection of a large extra-adrenal paraganglionoma. The tumour is secreting metanephrine. The most likely therapy to be commenced at the preassessment clinic prior to surgery is

a) Prazocin
b) Phentolamine
c) Magnesium
d) Phenoxybenzamine
e) Ca channel blocker

A

Phenoxybenzamine

UpToDate
Phenoxybenzamine​ is the preferred drug for preoperative preparation to control blood pressure and arrhythmia in most centers in the United States. It is an irreversible, long-acting, nonspecific alpha-adrenergic blocking agent.
With their more favorable side-effect profiles and lower financial cost, selective alpha-1-adrenergic blocking agents (eg, ​prazosin​, t​ erazosin​, or d​ oxazosin​) are utilized in many centers or are preferred to ​phenoxybenzamine​ when long-term pharmacologic treatment is indicated (eg, for metastatic pheochromocytoma).

How well did you know this?
1
Not at all
2
3
4
5
Perfectly
37
Q

23.1 The parameter that changes most with increasing age in the otherwise normal lung is the

a. Closing capacity
b. Residual volume
c. FRC
d. Lung capacity.

A

a) Closing capacity

see graph in Millers

How well did you know this?
1
Not at all
2
3
4
5
Perfectly
38
Q

20.2 The flow volume loop is most consistent with (Flow-volume loop shown)

a) Variable intra-thoracic obstruction
b) Variable extra-thoracic obstruction
c) Lower airway obstruction
d) Fixed upper Airway obstruction
e) Mixed pattern

A

a) Variable intra-thoracic obstruction

Dynamic (or variable, nonfixed) intrathoracic obstruction: flow limitation and flattening are noted on the expiratory limb of the loop.

How well did you know this?
1
Not at all
2
3
4
5
Perfectly
39
Q

20.2 This lung ultrasound shows

a) Normal lungs
b) Pulmonary odema
c) Pneumothorax
d) Pleural effusion
e) Pneumonia

A

a) Normal lungs

M-mode image demonstrating seashore sign seen with normal lung sliding.

How well did you know this?
1
Not at all
2
3
4
5
Perfectly
40
Q

23.1 An absolute contraindication to skin prick testing for the diagnosis of allergies is

a. Pregnancy
b. Severe dermatographia
c. Concurrent antihistamine use
d. Concurrent beta blocker
e. Asthma

A

b) severe dermatographia

How well did you know this?
1
Not at all
2
3
4
5
Perfectly
41
Q

23.1 An absolute contraindication to skin prick testing for the diagnosis of allergies is

a. Pregnancy
b. Severe dermatographia
c. Concurrent antihistamine use
d. Concurrent beta blocker
e. Asthma

A

b) severe dermatographia

How well did you know this?
1
Not at all
2
3
4
5
Perfectly
42
Q

23.1 Patients with rheumatoid arthritis and the most common form of atlantoaxial instability have a widened atlantodental interval. This is measured between the

A. distance from posterior surface of dens to anterior surface of posterior arch of atlas
B. distance from anterior surface of dens to anterior surface of posterior arch of atlas
C. distance from posterior surface of dens to anterior surface of anterior arch of atlas
D. distance from posterior surface of dens to posterior surface of posterior arch of atlas
E. distance from anterior surface of dens to posterior surface of anterior arch of atlas

A

E. distance from anterior surface of dens to posterior surface of anterior arch of atlas

The atlantodental interval is used in the diagnosis of atlanto-occipital dissociation injuries and injuries of the atlas and axis.

The anterior atlantodental interval is the horizontal distance between the posterior cortex of the anterior arch of the atlas (C1) and the anterior cortex of the dens in the median (midsagittal) plane

Normal values for anterior atlantodental interval are:
radiographs:
adults:
males: <3 mm
females: <2.5 mm 1 (although most authors describe <3 mm ref)

children:
<5 mm ref

CT: adults: <2 mm

How well did you know this?
1
Not at all
2
3
4
5
Perfectly
43
Q

21.1 Cardiovascular effects of hyperthyroidism include

a. Decreased SVR
b. Increased SVR
c. Decreased diastolic relaxation
d. Decreased PVR
e. Increased diastolic blood pressure

A

decreased SVR

Hyperthyroidism causes a hyperdynamic circulation, characterized by increased cardiac contractility and heart rate, increased preload, and decreased systemic vascular resistance (SVR), resulting in significantly increased cardiac output

https://www.sciencedirect.com/science/article/pii/S0735109718333795#:~:text=Hyperthyroidism%20causes%20a%20hyperdynamic%20circulation,in%20significantly%20increased%20cardiac%20output.

How well did you know this?
1
Not at all
2
3
4
5
Perfectly
44
Q

21.1 A 50-year-old man is seen prior to his hip revision surgery. His blood results are

Hb 110 (130-170 normal range)
Ferritin 31 (30-100 range)
Transferrin saturation 21% (normal 20-80)
CRP 10 (0.1-10 normal)

The most likely diagnosis is

a) iron deficiency anaemia
b) anaemia of chronic disease
c) anaemia of chronic inflammation
d) anaemia of chronic inflammation with iron deficiency
e) megaloblastic anaemia

A

Anaemia of chronic inflamation with iron deficiciency

How well did you know this?
1
Not at all
2
3
4
5
Perfectly
45
Q

22.1 A 50-year-old man with carcinoid syndrome having a resection of a peripheral hepatic metastasis develops a sudden fall in BP from 110/70 mmHg to 85/50 mmHg without significant bleeding. The most appropriate management is

a. Normal saline bolus
b. Octreotide 50mcg bolus
c. Metaraminol 0.5mg
d. Noradrenaline 5mcg bolus
e. Calcium 6.8mmol

A

b. Octreotide 50mcg bolus

Vasoactive hormone release intra-operatively is best treated with intravenous boluses of 20–50 µg of octreotide, titrated to haemodynamic response. Vasopressin as an alternative vasoconstrictor that may be useful if prolonged vasoconstriction is required; however, the evidence base is small.

It must be borne in mind that concomitant fluid losses, especially bleeding, may be responsible for intra-operative instability rather than hormone excess and that fluid resuscitation may be the answer rather than further octreotide therapy

https://academic.oup.com/bjaed/article/11/1/9/285683

How well did you know this?
1
Not at all
2
3
4
5
Perfectly
46
Q

22.1 Suxamethonium may be safely given to patients with

a. Becker muscular dystrophy
b. Cerebral palsy
c. Guillain Barre
d. Frederich’s ataxia
e. Duchenne muscular dystrophy

A

CP

b. Cerebral palsy
->sux and volatiles are not contraindicated
-> presence of extrajunctional receptors may cause hyperkalaemia

a. Becker muscular dystrophy
-> essentially milder Duchenne’s (see duchenne response to Sux)

b. Cerebral palsy
-> Sux and volatiles not contraindicated
-> reduced MAC requirement
-> increased sensitivity to muscle relaxants

c. Guillain Barre
-> sux contraindicated due to risk of hyperkalaemia
-> increased sensitivity to Non depolarising NB

d. Frederich’s ataxia
-> sux should be avoided due to risk of hyperkalaemia

e. Duchenne muscular dystrophy
-> sux and volatiles contraindicated due to rick of hyperkalaemia and rhabdomyolysis

How well did you know this?
1
Not at all
2
3
4
5
Perfectly
47
Q

21.1 Suxamethonium may be safely given to patients with

a) chronic spinal cord injury
b) Hypokalaemic periodic paralysis
c) muscular dystrophy
d) myasthenia gravis
e) multiple sclerosis

A

d) myasthenia gravis

In contrast to other neuromuscular disorders, succinylcholine may be used in myasthenia gravis. The required dose may need to be increased by up to two-fold, as those with the disease show a relative resistance to the drug.

Sux is not recommended in patients with neuromuscular disease due to:
1. presence of extrajunctional receptors and risk of hyperkalaemia and rhabodmyolysis
2. fasiculations causing temperomandibular muscle spasm preventing intubation

Suxamethonium is
contraindicated in patients with recent burns or
spinal cord trauma causing paraplegia (can be given
immediately after the injury, but should be avoided
from approximately day 10 to day 100 after the injury),

ED95 is 0.8mg/kg in a MG patient

How well did you know this?
1
Not at all
2
3
4
5
Perfectly
48
Q

21.2 You are examining the precordium of a patient in the preadmission clinic and hear a fourth heart sound at the apex. This finding is consistent with

a) AR
b) Athlete
c) Normal
d) Hypertension

A

) Hypertension

Talley & O’Connor CVS Exam:
S3: Physiological in pregnancy; sign of LV failure; AR & MR
S4: Never physiological, most often due to systemic hypertension

Atrial gallop - stiff LV
- hypertrophy or ischaemic ventricle

Source CV phys

How well did you know this?
1
Not at all
2
3
4
5
Perfectly
49
Q

21.1 A man who had successful treatment of a germ cell tumour 10 years ago presents for laparoscopic appendectomy. Your intraoperative management should consider

a) ETCO2 45
b) RR 20
c) MAP 90
d) SpO2 88–92%

A

d) SpO2 88–92%

oxygen administration/ low fio2
assumed bleomycin

Bleomycin
Bleomycin is a particularly important chemotherapy drug for the anaesthetist to be aware of. Bleomycin is often used to treat germ cell tumours and Hodgkin’s disease in a curative setting. The major limitation of bleomycin therapy is the potential for subacute pulmonary damage that can progress to life-threatening pulmonary fibrosis. Pulmonary toxicity occurs in 6–10% patients and can be fatal.2 Exposure to high-inspired concentration oxygen therapy, even for short periods, as experienced during anaesthesia, is often implicated in causing rapidly progressive pulmonary toxicity in patients previously treated with bleomycin.3 These claims have been considered controversial by some, but it is the authors’ recommendation that any patient previously exposed to bleomycin therapy should be treated as high risk, and summary guidance regarding oxygen therapy is shown in Table 4.

Summary guidance—oxygen therapy for patients who have received bleomycin > Patients have a life-long risk of bleomycin-induced lung injury
> Oxygen therapy should be avoided if at all possible
> Clinical procedures (and leisure activities) involving a high should be avoided If a patient is hypoxic
> O2 therapy should be minimized to maintain O2 saturation of 88–92%
> High oxygen concentrations should be used with extreme caution for immediate life-saving indications only (to maintain O2 saturation of 88–92%)

How well did you know this?
1
Not at all
2
3
4
5
Perfectly
50
Q

22.2 Created by the Global Initiative for Chronic Obstructive Lung Disease (GOLD 2017), the numerical GOLD classes 1 to 4 are classes of severity for chronic obstructive pulmonary disease (COPD). These classes are based on an assessment of the

A. Exertional dyspnoea
B. Exertional dyspnoea and FEV1
C. Exertional dyspnoea and number of exacerbations per year
D. Spirometry FEV1 only
E. Number of exacerbations per year only

A

ALTERED 22.1 QUESTION

D Spirometry FEV1 only

GOLD 1 > 80% Pred
GOLD 2 50-79% Pred
GOLD 3 30-49% Pred
GOLD 4 < 30% Pred

How well did you know this?
1
Not at all
2
3
4
5
Perfectly
51
Q

21.1 In elderly patients without diabetes mellitus the use of aspirin in primary prevention of disease

a. Reduced cardiovascular mortality
b. Increased incidence of major bleeding
c. Increased cancer related death
d. Lower all cause mortality
e. Reduced thromboembolic events

A

increased incidence of major bleeding

52
Q

21.2 A man who had successful treatment of a germ cell tumour ten years ago presents for laparoscopic appendectomy. Your intraoperative management should consider

a) Lung protective ventilation
b) Oncoanaesthesia
c) Lowest FiO2 possible
d) MAP 60

A

c) Lowest FiO2 possible

Bleomycin

Bleomycin is a particularly important chemotherapy drug for the anaesthetist to be aware of.
Bleomycin is often used to treat germ cell tumours and Hodgkin’s disease in a curative setting.
The major limitation of bleomycin therapy is the potential for subacute pulmonary damage that can progress to life-threatening pulmonary fibrosis.
Pulmonary toxicity occurs in 6–10% patients and can be fatal.
Exposure to high-inspired concentration oxygen therapy, even for short periods, as experienced during anaesthesia, is often implicated in causing rapidly progressive pulmonary toxicity in patients previously treated with bleomycin.
These claims have been considered controversial by some, but it is the authors’ recommendation that any patient previously exposed to bleomycin therapy should be treated as high risk, and summary guidance regarding oxygen therapy is shown:

53
Q

22.2 A drug that is contraindicated for a patient with a history of heparin-induced thrombocytopaenia is

a) Bivalirudin
b) Danaparoid
c) Prothrombinex
d) Fib conc

A

c) Prothrombinex

Has factors 2, 9, 10, heparin, ATIII

54
Q

21.1, 21.2 The most common cause of cor pulmonale is

a) Shunt
b) COPD
c) Volume overload
d) PE
e) Kyphoscoliosis

A

Chronic obstructive pulmonary disease (COPD) is the most common cause of cor pulmonale

leads to an increase in RV afterload secondary to changes in pulmonary vascular structure and mechanics, and lung hyperinflation.

Patients with COPD who subsequently develop RV dysfunction have an increased risk of admission to hospital and mortality

55
Q

22.1 A 24-year-old man with Wolff-Parkinson-White syndrome is having anaesthesia for a knee arthroscopy. During the procedure he develops the following rhythm. His blood pressure is 100/65mmHg.
The most appropriate treatment is

a. Adenosine
b. Procainamide
c. Verapamil

A

b. Procainamide
BJA: Perioperative cardiac arrhythmias
https://academic.oup.com/bja/article/93/1/86/265716

  • Paroxysmal SVT (PSVT) due to re‐entrant circuits that involve accessory pathways (congenital electrical connections between the atrium and ventricle that bypass the AV node, such as Wolff–Parkinson–White Syndrome) pose caveats in the management of SVT.
  • It should be noted that patients with accessory pathways, in addition to PSVT, may also develop atrial fibrillation, and in the latter situation are at increased risk for developing ventricular fibrillation (VF) upon exposure to classic AV‐nodal blocking agents (digoxin, calcium channel blockers, beta blockers, adenosine) because these agents reduce the accessory bundle refractory period.
  • In such cases, i.v. procainamide, which slows conduction over the accessory bundle, is an acceptable option. Flecainide and amiodarone should also be considered, and cardiology consultation may be helpful.2
56
Q

21.1 Of the following, the lifestyle modification that is least effective in reducing essential hypertension is

a. Stopping caffeine
b. Low sodium diet
c. Low potassium diet
d. Exercise

A

c. Low potassium diet (high K diet - dec HTN)

57
Q

21.2 In pulmonary function testing the presence of airflow limitation is defined by a post- bronchodilator FEV1/FVC ratio less than
a) 0.5
b) 0.6
c) 0.7
d) 0.8

A

c) 0.7

58
Q

20.1 A patient with von Willebrand deficiency Type 1 presents with mild but persistent epistaxis.

First-line medical therapy should include:

a) Factor VII
b) Factor VIII
c) Recombinant von Willebrand factor
d) TXA
e) FFP

A

d) TXA

VWD Types:
1 - quantitative - minor effect on bleeding - DDAVP useful
2 - qualitative - spectrum of effects on bleeding - (2a,2b,2m,2n) - DDAVP may be useful in consult with haem
3 - absence - major bleeding - no effect of DDAVP

factors not recommended in Type 1
TXA and DDAVP are recommended but DDAVP not in list
TXA 10mg/kg IV q8h
DDAVP 300mcg intranasal 90-120 mins preop
(DDAVP increases factor VIII levels 2-5x via release of VWF which binds VIII and prevents its clearance)

Treatment of bleeding in an individual with von Willebrand disease (VWD) depends on:
1. Severity of bleeding
2, Site of bleeding
3. the type of VWD
4. the previous responses to therapy.

The two main approaches:
1. Increasing the level of normal von Willebrand factor (VWF) activity via DDAVP
2. Replacing the defective VWF with VWF concentrates

VWF concentrates have been demonstrated to provide excellent to good hemostasis in a number of patient populations and a number of bleeding types.

DDAVP is only effective in some individuals, produces a smaller increase in VWF activity, and has a later onset and shorter duration of action.

59
Q

22.2 A 56-year-old patient presents with exertional syncope. The most likely diagnosis is
(previously this was a 26yo)

a) HOCM
b) Aortic stenosis
c) Long QT syndrome

A

b) Aortic Stenosis

60
Q

22.2 Unsupported ventilation in a non-anaesthetised patient with long-standing tetraplegia is improved when

a) Left lateral
b) Right lateral
c) Supine
d) Trendelenberg
e) Reverse Trendelenberg

A

d) Trendelenberg c) Supine

Moving from upright to supine affects the respiratory function of the tetraplegic and high paraplegic individual differently to the able-bodied person.

The increase in abdominal girth when sitting in tetraplegia is secondary to decreased abdominal muscle strength and the associated increased abdominal wall compliance.

In the seated position, the abdominal contents are less supported by the decreased abdominal wall muscle tone and fall forward, increasing the waist size and lowering the diaphragm.

In able-bodied subjects, the FVC is reduced in the supine position, whereas in tetraplegia it is increased.

Postural changes are associated with symptoms; patients with an acute, high SCI report less breathlessness when supine compared to sitting.

In the supine position, the weight of the abdominal contents forces the diaphragm to a higher resting level so that contraction produces greater absolute excursion of the diaphragm; an effect that can be increased when the person with tetraplegia is tipped 15° head down from supine such that the vital capacity rises by a further 6%

61
Q

20.2 The flow volume loop is most consistent with (Flow-volume loop shown)

a) Variable intra-thoracic obstruction
b) Variable extra-thoracic obstruction
c) Mixed Pattern
d) Fixed upper Airway obstruction
e) Normal

A

e) Normal

Normal flow-volume loop: the expiratory portion of the flow-volume curve is characterized by a rapid rise to the peak flow rate, followed by a nearly linear fall in flow. The inspiratory curve is a relatively symmetrical, saddle-shaped curve.

62
Q

20.2 The therapy most likely to decrease mortality in neonates with congenital diaphragmatic hernia is

a) Early surgical intervention - within 6 hours
b) HFOV
c) Lung protective ventilation
d) Nitric oxide
e) thoracoscopic vs open approach?

A

c) Lung protective ventilation

A congenital diaphragmatic hernia (CDH) occurs when a defect in the diaphragm allows abdominal organs to protrude into the thoracic cavity (Fig. 1). It affects approximately 1 in 3600 registered births and is a potentially life-threatening condition, the severity of which is primarily related to the extent of lung hypoplasia and pulmonary hypertension.

Advances in management strategies include protective ventilation, careful timing of surgery, the judicious use of extracorporeal membrane oxygenation (ECMO) and the introduction of both thoracoscopic and fetal intervention, but it remains a challenging condition to treat successfully with overall mortality rates still around 30%

A validated scoring system has been proposed based on various measurements, including:

(i) birth weight <1.5 kg,
(ii) Apgar score at 5 mins <7,
(iii) presence of chromosomal abnormality,
(iv) presence of major cardiac abnormality, and
(v) suprasystemic pulmonary hypertension on echocardiography.

This scoring system enables patients to be stratified into low (<10%), intermediate (∼25%) or high risk (∼50%) mortality groups.
Cardiac defects have been shown to worsen outcome regardless of the severity of the hernia
The presence of a small contralateral lung or a bilateral CDH are also poor prognostic signs.

Ventilation
Historically, aggressive ventilation was used to induce hypocapnia and alkalosis and thereby reduce pulmonary hypertension; however, protective ventilation strategies that avoid further injury to damaged lung tissue have reduced mortality in CDH. The CDH EURO Consortium advocates aiming for the limitation of peak inspiratory pressures to 25 cm H2O with PEEP kept at 3–5 cm H2O and allowing permissive hypercapnia.

High-frequency oscillatory ventilation
High frequency oscillatory ventilation (HFOV) is classically used as a rescue strategy when hypoxia and severe hypercapnia persist despite maximal conventional ventilation. The VICI trial (2016) randomised 171 neonates to conventional ventilation or HFOV as the initial mode of ventilation and found no significant difference in mortality, but those who had conventional ventilation were ventilated for shorter periods, needed less nitric oxide, sildenafil and ECMO, and had lower requirements for inotropic drugs.

Timing of surgery
Historically, CDH repair was treated as a surgical emergency. However, the degree of pulmonary hypoplasia is the major influence on prognosis and emergency surgery therefore confers little benefit. There is much debate but little consensus within the literature regarding the optimal timing of surgery.
Recommendations from the CDH EURO Consortium state that the following physiological parameters should be met before surgery:

(i) mean arterial pressure normal for gestation,
(ii) preductal oxygen saturation consistently 85–95% on FiO2 <0.5,
(iii) lactate below 3 mmol litre−1, and
(iv) urine output more than 1 ml kg−1 h−1 12

These recommendations do, however, acknowledge that repair on ECMO is a viable treatment strategy in the context of appropriate patient selection.

Little evidence Thorascopic approach has improved mortality compared to open approach

BJA Education Article - ​Anaesthetic management of patients with a congenital diaphragmatic hernia
https://www.bjaed.org/article/S2058-5349(18)30013-1/fulltext

63
Q

22.1 A patient’s glomerular filtration rate is estimated at 35 mL/min/1.73m2. The patient’s chronic kidney disease can be classified as Stage

a. 5
b. 4
c. 3a
d. 3b
e. 2

A

3b

Category GFR
ml/min/1.73 m2 Terms
G1 ≥90 Normal or high
G2 60-89 Mildly decreased*
G3a 45-59 Mildly to moderately decreased
G3b 30-44 Moderately to severely decreased
G4 15-29 Severely decreased
G5 <15 Kidney failure

Assign Albuminuria category as follows:
Albuminuria categories in CKD
Category ACR (mg/g) Terms
A1 <30 Normal to mildly increased
A2 30-300 Moderately increased*
A3 >300 Severely increased**
Abbreviations: ACR, albumin-to-creatinine ratio; CKD, chronic kidney disease.
*Relative to young adult level.
**Including nephrotic syndrome (albumin excretion ACR >2220 mg/g)

**Collectively referred to as “CGA Staging”

64
Q

21.1 A 50 year old man has the following pulmonary function test result. The most consistent diagnosis is

FEV1 - test result - predicted - % predicted 68%
FVC - test result - predicted - % predicted 68%
DLCO 46%

a) Asthma
b) Myasthenia Gravis
c) Emphysema
d) Sarcoidosis
e) Pulmonary Hypertension

A

d) Sarcoidosis

Pulmonary hypertension: Normal spirometry + low DLCO
Asthma: obstructive pattern and normal DLCO
Obesity: restrictive pattern and normal DLCO
Sarcoid: restrictive pattern and low DLCO

65
Q

20.2 Prothrombinex VF is a factor concentrate. It is indicated for the management of bleeding caused by

a Von Willebrand disease
b Haemophilia a
c Haemophilia b
d Haemophilia c
e Congenital fibrin deficiency

A

c Haemophilia b

66
Q

20.1 A patient with RA has been on 5mg of prednisone long term and is coming in for a joint replacement what is the appropriate management of their steroids?

a) 5mg oral pred
b) 10mg oral pred
c) No steroids
d) 50mg hydrocortisone IV
e) 100mg hydrocortisone IV

A

e) 100mg hydrocortisone IV
> note: alternatively, 6-8mg dexamethasone IV would suffice

67
Q

23.1 Suxamethonium may be safely given to patients with

a. Becker muscular dystrophy
b. Friedreich’s ataxia
c. Guillain-Barre
d. Cerebral palsy
e. Duchenne muscular dystrophy

A

d) myasthenia gravis
or
d) Cerebral palsy
->sux and volatiles are not contraindicated
-> presence of extrajunctional receptors may cause hyperkalaemia

if responses remembered incorrectly but of this list CP is probably the answer

a. Becker muscular dystrophy
-> essentially milder Duchenne’s (see duchenne response to Sux)

b. Cerebral palsy
-> Sux and volatiles not contraindicated
-> reduced MAC requirement
-> increased sensitivity to muscle relaxants

c. Guillain Barre
-> sux contraindicated due to risk of hyperkalaemia
-> increased sensitivity to Non depolarising NB

d. Frederich’s ataxia
-> sux should be avoided due to risk of hyperkalaemia

e. Duchenne muscular dystrophy
-> sux and volatiles contraindicated due to rick of hyperkalaemia and rhabdomyolysis

In contrast to other neuromuscular disorders, succinylcholine may be used in myasthenia gravis. The required dose may need to be increased by up to two-fold, as those with the disease show a relative resistance to the drug.

Sux is not recommended in patients with neuromuscular disease due to:
1. presence of extrajunctional receptors and risk of hyperkalaemia and rhabodmyolysis
2. fasiculations causing temperomandibular muscle spasm preventing intubation

REPEAT

68
Q

20.2 Apert syndrome is associated with

A) Atlanto-occipital instability
B) Hypotonia
C) Increased ICP
D) hypercalcemia
E) Mucopolysaccharoidosis

A

Raised ICP
*also associated with a difficult airway (Difficult BMV Ventilation)

Apert syndrome:
Autosomal dominant abnormality of first branchial arch causing premature closure of cranial sutures, midface hypoplasia, choanal atresia, cleft palate, fusion of cervical spine (mainly C5-C6) and syndactyly.

May have associated cardiac and renal abnormalities as well as intellectual impairment due to megalocephaly, hypoplasia of white matter and agenesis of the corpus callosum.

Obstructive sleep apnea is present in 50% and there may be an increased incidence of upper airway obstruction at induction, which is mostly overcome by routine maneuvers.

Classically, craniosynostosis release with fronto-orbital advancement is completed at 6 to 12 months of age if intracranial pressure (ICP) is normal [​24-26​]. However, elevated ICP may occur in up to 43 percent of cases. In this event, prompt surgical advancement and potentially ventriculoperitoneal shunt placement is required

69
Q

22.2 The drug of choice for the treatment of duct-dependent congenital heart disease is

a) Sildenafil
b) Prostacyclin
c) Carboprost
d) Alprostadil
e) NSAID

A

d) Alprostadil

https://www.rch.org.au/piper/neonatal_medication_guidelines/Alprostadil_(Prostin_VR)%E2%80%93(Prostaglandin_E1)/

Alprostadil (PROSTAGLANDIN E1) is a synthetic prostaglandin used to relax the ductus arteriosus in early post-natal life, where a patent ductus is critical for survival, including Tetralogy of Fallot, pulmonary atresia, pulmonary stenosis, tricuspid atresia and transposition of the great arteries.

Dose
To open a closed ductus arteriosus:
0.1 micrograms/kg/minute (100 nanograms/kg/min). An effect is usually seen within 30-60 minutes. Reduce the dose once an effect is seen or as directed by a Consultant.1

Doses > 0.1 micrograms/kg/minute are rarely more effective and may cause serious adverse effects.3

To maintain patency of ductus arteriosus:
0.01 to 0.02 micrograms/kg/minute (10-20 nanograms/kg/min).1, 2

For persistent pulmonary hypertension of the newborn (PPHN):
0.01 to 0.05 micrograms/kg/minute (10-50 nanograms/kg/min).2

70
Q

22.1 A 60-year-old woman presents for thrombectomy with left lower leg ischaemia. She has not received any medications since presentation, and takes none at home. The sole abnormality on laboratory testing is an activated partial thromboplastin time (APTT) of 52 seconds. The most likely cause of the raised APTT is

a. Lupus anticoagulant
b. Erroneous reading
c. Cold agglutinins
d. Factor VII deficiency
e. Haemophilia A

A

a. Lupus anticoagulant

Factor VII
-> prolonged PT but not APTT

Cold Agglutinins
-> prolonged PT and APTT
-> “sole abnormality”

Haemophilia A
-> isolated prolonged APTT
-> associated with bleeding and not clotting

Lupus Anticoagulation
-> increased risk of clotting
-> prolonged APTT and normal PT

71
Q

23.1 In a 20-year-old with cystic fibrosis, the most likely finding on pulmonary function
tests is

a. Mixed obstruction and restrictive pattern
b. Restrictive with normal DLCO
c. Restrictive with low DLCO
d. Obstruction with reduced RV
e. Obstructive with reduced FEV1

A

e. Obstructive w/ reduced FEV1

Mucous narrowing airways = obstructive
Parenchymal damage = restrictive

Obstructive PFP remains the most common pulmonary function pattern in adult CF and is associated with
-decrease FEV1 & FVC/FEV1

For patients with CF, an obstructive pattern is generally seen, with a decrease in forced expiratory volume in 1 s (FEV1), and forced vital capacity (FVC) to FEV1 ratio.

https://academic.oup.com/bjaed/article/11/6/204/263786

72
Q

21.1 A third heart sound at the apex may be heard with

a) pulmonary stenosis
b) pulmonary hypertension
c) pericarditis
d) pregnancy

A

d) pregnancy

A third heart sound reflects rapid left ventricular distention along with an increased atrioventricular flow

Heard in Congestive heart failure

Associated with Dilated Cardiomyopathy with dilated ventricles

Less commonly valvular regurgitation and left to right shunts

May be normal physiological finding in patients less than 40yrs old

73
Q

21.2 Painless post-operative visual loss with preserved pupillary reflexes is most likely due to

a) Retinal detachment
b) Anterior ischaemic optic neuropathy
c) Corneal abrasion
d) Posterior ischaemic optic neuropathy
e) Posterior cerebral ischaemia

A

PCA

e) Posterior cerebral ischaemia

UTD: Postoperative visual loss after anaesthesia for nonocular surgery

Pupillary light reflexes*
Unilateral central retinal artery occlusion, ischemic optic neuropathy, and retrobulbar hematoma result in a poor or absent pupillary response to light (“direct” response) with a normal response when light is directed to the other pupil (“indirect” response); this “relative afferent pupillary defect” is revealed when tested with the swinging flashlight maneuver; if these processes are bilateral, there will be poor or absent direct pupillary responses and a relative afferent pupillary defect only if asymmetric.
Mid-dilated and nonreactive pupils are consistent with acute angle-closure glaucoma, while sluggish to fixed and dilated pupils are seen with glycine-induced visual loss.
Pupillary light reflexes are normal in cases of corneal abrasion, cerebral or cortical visual loss, and in cases of PRES. Examination of pupils is discussed more fully separately.

74
Q

21.1 The intubating dose of atracurium in a patient with post-polio syndrome should be

a. 10 %
b. 20
c. 50
d. 100
e. 200

A

c. 50%
0.25mg/kg (Half)

Source: PolioSA

And ANZCA bulletin 2015 ‘Anaesthetists need to be wary of post polio syndrome’ -“twice as sensitive to non-depolarising muscle relaxants”

75
Q

20.2 A 50 year old man has the following pulmonary function test result.

FEV1 68% predicted,
FVC 68% predicted,
DLCO 46% predicted

The most consistent diagnosis is

a) Asthma
b) Myasthenia Gravis
c) Emphysema
d) Sarcoidosis
e) Pulmonary Hypertension

A

Repeat

d) Sarcoidosis

Normal FEV1/FVC ratio = no obstruction
Low FVC = restrictive pattern
Low DLCO = interstitial lung disease

Asthma and emphysema would have obstructive pattern.
Myasthenia gravis would have normal DLCO
Pulmonary HTN would have normal spirometry and low DLCO.

76
Q

22.2 A 25-year-old male has continued postoperative bleeding after an extraction of an impacted third molar tooth under a general anaesthetic. The patient mentions that his father bruises quite easily. His coagulation screen reveals: (provided). The most likely diagnosis is
(APTT raised, PT normal?)

a. Factor V leiden
b. haemophilia A
C. Von willebrand’s disease
D. Haemophilia B

A

b. von willebrand’s disease

  • autosomal dominant inheritance
  • may have normal or prolonged APTT, PT is normal

*Haem A: X-linked recessive disorder; would expect prolonged aPTT, and normal PT
*Haem B: X-linked recessive disorder; would expect normal aPTT and normal PT

REPEAT

vWD can have prolonged APTT or normal APTT. Haemophilias are X-linked

77
Q

23.1 A diagnosis of metabolic syndrome is NOT supported by

A. Impaired glucose tolerance
B. High HDL
C. Obesity
D. High triglycerides
E. Hypertension

A

b. high HDL-C

https://www.ahajournals.org/doi/10.1161/CIRCULATIONAHA.105.169405

78
Q

20.1 In a patient with known COPD, which of the following post bronchodilator spirometry results is consistent with a GOLD 3 classification? (Global initiative for chronic Obstructive Lung Disease)

a) FEV1 83%
b) FEV1 57%
c) FEV1 43%
d) FEV1 27%
e) FEV1 19%

A

c) FEV1 43%

In pulmonary function testing, a post-bronchodilator FEV1/FVC ratio of <0.70 is commonly considered diagnostic for COPD. The Global Initiative for Chronic Obstructive Lung Disease (GOLD) system categorises airflow limitation into stages. In patients with FEV1/FVC <0.70:

GOLD 1 - mild: FEV1 ≥80% predicted

GOLD 2 - moderate: 50% ≤ FEV1 <80% predicted

GOLD 3 - severe: 30% ≤ FEV1 <50% predicted

GOLD 4 - very severe: FEV1 <30% predicted.
79
Q

22.2 Drug classes demonstrated to reduce mortality in chronic heart failure with reduced ejection fraction include all of the following EXCEPT

A. ACE inhibitors
B. Beta blockers
C. Angiotensin receptor blockers
D. Spironolactone
E. Digoxin

A

Digoxin

80
Q

21.2 A 45-year-old man has the following results on his blood biochemistry testing: The most likely diagnosis is

  • Bili 30*
  • AST 1000*
  • ALT 500*
  • Albumin 30*
    *These blood results are not the original stem.

The most likely diagnosis is:

a) Hepatitis
b) Alcoholic liver disease
c) Paracetamol toxicity
d) Cholecystitis

A

b) Alcoholic liver disease
- AST>ALT

In hepatitis and paracetamol toxicity would expect ALT>AST.

In cholecystitis, would expect a cholestatic picture with raised conjugated bilirubin and raised GGT/ALP.

LITFL: Overall analysis of Liver Function Tests (LFT)

Transaminitis: Aminotransferases (AST, ALT)
- Generally associated with hepatocellular damage
- Generally not associated with cholestasis

Ratio of AST and ALT can be useful in differential
ALT is more specific for liver damage than AST

AST: ALT =1
-> Associated with ischaemia (CCF and ischaemic necrosis and hepatitis)

AST: ALT >2.5
-> Associated with Alcoholic hepatitis
-> Alcohol induced deficiency of pyridoxal phosphate
AST: ALT <1
-> High rise in ALT specific for Hepatocellular damage
-> Paracetamol OD with hepatocellular necrosis
-> Viral hepatitis, ischaemic necrosis, toxic hepatitis
-> Elevation with cholestasis (ALP, GGT)

ALP – primarily associated with cholestasis and malignant hepatic infiltration
Marker of rapid bone turnover and extensive bony metastasis

GGT – sensitive to alcohol ingestion
Marker of hepatocellular damage but non-specific
Sharpest rise associated with biliary and hepatic obstruction

81
Q

22.2 The influence of end-stage renal disease on the plasma clearance and dose of sugammadex is that the

a) Increased clearance – increased dose
b) Decreased clearance – reduced dose
c) Decreased clearance – same dose
d) No change in clearance or dose

A

c) Decreased clearance – same dose

82
Q

21.2 The drug of choice for the treatment of duct dependent congenital heart disease is

a) Alprostadil
b) Prostacyclin
c) Carboprost
d) Sildenafil
e) NSAID

A

a) Alprostadil

Prostin (PGE1)

83
Q

20.2 Severe obstructive sleep apnoea in adults is confirmed if during polysomnography if the apnoea/hypopnea index (AHI) is greater than or equal to

A) 10
B) 20
C) 30
D) 40
E) 50

A

C) 30

84
Q

22.1 Ehlers-Danlos Syndrome is associated with each of the following EXCEPT

a. Blood vessel fragility
b. LA resistance
c. Intellectual impairment
d. Glaucoma

A

Intellectual impairment

No solid refs

85
Q

20.2 A 55-year-old patient who has undergone trans-sphenoidal hypophysectomy for a growth-hormone secreting adenoma has a urine output of one litre in the first postoperative hour. The following results are obtained. The most appropriate early management is

Na 145, Urinary osm ~200, Serum Osmolarity ~320

a) DDAVP
b) Hypertonic saline
c) Normal Saline 1 L bolus
d) 100 ml/hr of saline
e) Fluid restrict

A

a) DDAVP

Polyuria
Low urine osm
High serum osm
High Na
post transsphenoidal sx
= Central DI

86
Q

23.1 The causes of macrocytic anaemia include

A. Liver failure
B. Renal failure
C. Thalassaemia
D. Thyrotoxicosis
E. Vitamin e deficiency

A

A

A - Alcohol is a common cause of macrocytosis and macrocytic anemia. (UpToDate)
B - No - normally normocytic chronic disease anaemia
C - No - microcytic
D - I can’t find anything on macrocytosis with thyrotoxicosis, but hypothyroidism definitely does
E - Possibly…. https://hemonc.mhmedical.com/content.aspx?bookid=1783&sectionid=121720217

87
Q

22.1 An adult male patient has a haemoglobin level of 80 g/L and his blood film shows a reticulocyte count of 10%. These findings are compatible with

a. ALL
b. Spherocytosis
c. Aplastic anaemia
d. Pernicious anaemia
e. Anaemia of chronic disease

A

Hereditary spherocytosis.

Auto-haemolytic, intraplenic haemolysis. High reticulocyte count (6-20%) (normal range 0.5-2%)

88
Q

22.1 A 72-year-old female smoker with hypertension presents to the emergency department with a wrist fracture after a fall. She has been increasingly tired and confused over the previous week. Her serum and urine electrolytes are (supplied). The most likely diagnosis is

(Low K, low Na, Normal Ur and Cr, Ur sodium <10mmol/L)

a. SIADH
b. Addison’s
c. Diuretic

A

Uncertain

SIADH:
1. hypotonic hyponatraemia
2. urine osmolality > plasma osmolality (<275mOsm/kg) (i.e. concentrated urine despite hypotonic blood)
3. urinary Na+ > 20mmol/L
4. normal renal, hepatic, cardiac, pituitary, adrenal and thyroid function
5. euvolaemia (absence of hypotension, hypovolaemia, and oedema)
6. correction by water restriction

Addison’s
Hypo natraemia
HYPER kalaemia
Hypo glycaemia
Acidosis

Diuretics
Hypo natraemia
Hypo kalaemia
High urinary Na and K

https://www.derangedphysiology.com/files/Electrolyte%20Disturbance.pdf

JAMA article on hyponatraemia
https://emergencymed.org.il/wp-content/uploads/2022/08/jama_adrogu_2022_rv_220011_1657919726.49616.pdf

89
Q

21.2 The diffusing capacity of the lungs for carbon monoxide (DLCO) is likely to be decreased with

a) Sarcoidosis
b) Asthma
c) Obesity
d) Pulmonary haemorrhage

A

sarcoid

90
Q

21.1 An ECG abnormality which is NOT usually associated with severe anorexia nervosa is

a) QT prolongation
b) TWI
c) ST depression
d) prolonged PR interval
e) sinus tachycardia

A

e) sinus tachycardia

https://academic.oup.com/bjaed/article/9/2/61/299563

Typically anorexic patients are hypotensive and bradycardic.
Bradycardia reflects the decrease in basal metabolic rate that arises as an adaptive response to starvation
Electrocardiographic abnormalities are common and may be found in >80% of strict dieters. These include:
1. atrioventricular block
2. ST depression
3. T wave inversion
4. QT prolongation.

91
Q

20.2 International guidelines state that patients presenting for major surgery have inadequate or low iron stores if their serum ferritin level is less than

a. 20 mcg/L
b. 30 mcg/L
c. 40 mcg/L
d. 50 mcg/L
e. 100 mcg/L

A

E. 100 mcg/L

?? < 30mcg/L

< 100mcg/L IF CRP > 5 and/or Transferrin saturation < 20

https://associationofanaesthetists-publications.onlinelibrary.wiley.com/doi/10.1111/anae.13773#:~:text=Recommendations%20for%20best%20clinical%20practice,-Physicians%20should%20consider&text=The%20presence%20of%20anaemia%20should,identification%20of%20absolute%20iron%20deficiency.

92
Q

21.2 A patient with known type 3 von Willebrand disease presents with persistent epistaxis. First-
line medical therapy should include

a) DDAVP
b) Prothrombin X
c) Factor VIIa
d) Factor VIII

A

TXA

DDAVP for T1
Factor 8 for T2/3 or unresponsive DDAVP
(RCH Guidelines)

93
Q

22.2 The curve labelled ‘b’ is most likely to represent the flow–volume loop of a patient with

a) Asthma
b) Post lung transplant
c) Pulmonary fibrosis
d) Tracheal stenosis
e) VC palsy

A

Tracheal stenosis

94
Q

21.1 A 30-year-old previously healthy woman is four days post-caesarean section. You are asked to see her to manage her abdominal pain. Over the last two days she has had increasing abdominal pain, increasing abdominal distension, tachycardia and nausea. An abdominal x-ray shows a caecal diameter of 9 cm. After excluding mechanical obstruction, an appropriate management option is

a) neostigmine infusion
b) morphine PCA
c) Naloxone
d) Lactulose

A

a) neostigmine infusion

Consider this Ogilve’s Syndrome
Psuedo-obstruction.
If > 9cm dilation, would need surgical management.

https://www.ncbi.nlm.nih.gov/pmc/articles/PMC3168359/#!po=17.5000

95
Q

21.1 A patient has bipolar disorder and is on long term lithium therapy. An analgesic which should be avoided is

a. Diclofenac
b. Tramadol
c. Oxycodone
d. Methadone

A

a. Diclofenac

LIthium perioperative concerns:
- Prolongation of NMB
- Reduction in anaesthetic agent requirement
- Avoid NSAIDs
- No withdrawl symptoms
- Discontinue 24hrs before surgery

NSAIDs differentially alter lithium concentrations by multiple mechanisms, and one of these is to reduce prostaglandin E2

BJA: perioperative advice for psychotropic drugs

96
Q

21.1 A condition or therapy that is NOT a contraindication to hyperbaric oxygen therapy is

A. Bleomycin
B. Cisplatin
C. Preterm neonate
D. Cerebral Abscess

A

D. Cerebral Abscess

HBOT Indications:
- Indications are related to need for enhanced Pressure or oOxygenation and treatment of infection:

Pressure
- air or gas embolism
- Decompression sickness

Oxygenation:
- arterial insufficiencies
(central retinal artery occlusion, enhancement of healing in wound problems)
- Carbon monoxide poisoning
- Compromised grafts and flaps
- Acute traumatic ischaemia
- Delayed radiation injuries
- Sudden sensorineural hearing loss
- Severe Anaemia
- Thermal burns

Infection:
- Clostridium myonecrosis (gas gangrene)
- Intracranial abscess
- Necrotising soft tissue infections
- Refractory osteomyelitis

Absolute Contraindications to HBOT:
- untreated PTx
- Premature Infants
- Bleomycin
- Disulfiram (antabuse)
- Cisplatin

Relative contraindications:
- Pregnancy
- Asthma
- Thoracic Surgery
- Emphysema with CO2 retention
- upper respiratory tract infections
- History of middle ear surgery or disorder
- History of seizures
- Fevers
- Congenital spherocytosis
- Optic neuritis

97
Q

21.1 The most common cause of mortality in children with diabetic ketoacidosis is

a. Cerebral oedema
b. Septic shock
c. Central pontine myelinolysis

A

a. Cerebral oedema

Cerebral Oedema

Source: UpToDate

98
Q

22.1 According to the Third International Consensus Definitions for Sepsis and Septic Shock (Sepsis-3), sepsis is defined as

a. SIRS criteria
b. Life threaning organ dysfunction with vasopressor requirement to maintain MAP >65 and lactate >2
c. Life threatening organ dysfunction caused by a dysregulated host response to infection
d. sBP <100, RR>22, altered mentation

A

Life threatening organ dysfunction caused by a dysregulated host response to infection

99
Q

23.1 A patient’s glomerular filtration rate is estimated at 35 mL/min/1.73m2. The patient’s chronic kidney disease can be classified as Stage

a. 5
b. 4
c. 3a
d. 3b
e. 2

A

Category GFR
ml/min/1.73 m2 Terms
G1 ≥90 Normal or high
G2 60-89 Mildly decreased*
G3a 45-59 Mildly to moderately decreased
G3b 30-44 Moderately to severely decreased
G4 15-29 Severely decreased
G5 <15 Kidney failure

Assign Albuminuria category as follows:
Albuminuria categories in CKD
Category ACR (mg/g) Terms
A1 <30 Normal to mildly increased
A2 30-300 Moderately increased*
A3 >300 Severely increased**
Abbreviations: ACR, albumin-to-creatinine ratio; CKD, chronic kidney disease.
*Relative to young adult level.
**Including nephrotic syndrome (albumin excretion ACR >2220 mg/g)

**Collectively referred to as “CGA Staging”

REPEAT

100
Q

21.1 A man who had successful treatment of a germ cell tumour 10 years ago presents for laparoscopic appendectomy. Your intraoperative management should consider

a) Lung protective ventilation
b) Oncoanaesthesia
c) Lowest FiO2 possible
d) MAP 60

A

c) Lowest FiO2 possible

oxygen administration/ low fio2
assumed bleomycin

101
Q

22.2 A raised (> 140% predicted) single-breath diffusing capacity of the lung for carbon monoxide (DLCO) can be caused by

a. Emphysema
b. COPD
c. interstitial lung disease
d. Asthma
e. Sarcoidosis

A

d. Asthma

What are the causes of an elevated DL CO ?

The causes of an elevated DLCO are numerous, but is most commonly caused by asthma and obesity (increased pulmonary blood flow). Pulmonary hemorrhage is an additional important cause.

https://www.atsjournals.org/doi/pdf/10.1513/AnnalsATS.201605-355CC

102
Q

21.1 In the treatment of persistent mucosal bleeding in patients with von Willebrand disease Type 3, Desmopressin (DDAVP) is

a) contraindicated due to risk of thrombocytopenia
b) indicated if previous response documented
c) indicated to improve plt function
d) contraindicated as it won’t work

A

d) contraindicated as it won’t work

Type 1:
-Quantitative defect of VWF

Type 2:
-Qualitative Defect of VWF
-Type 2 subclassification depending on plt binding function, F8 binding capcacity, number of high molecular weight VWF multimers

Type 3:
- complete absence of VWF

Treatment:
- do not need blood components to control haemorrhage
-F8 plasma concentration >100 for major surgery and >50 for minor surgery
-DDAVP approved for use in Type 1, no use in type 3, discuss its use with haematology in type 2 due to its variable effect
-DDAVP given atleast 90mins before operation
-TXA may be useful
-VWF/F8 concentrates indicated in severe cases, type 3 and qualitiative defects in VWF
-Plt infusions should be considered in persistent bleeding
-Cryo has an unpredictable effect, only used if other treatments have failed

103
Q

21.2 A 50 year old man has the following pulmonary function test result:

FEV1 98% predicted
FVC 95% predicted
DLCO 43% predicted

The diagnosis is most consistent with:

a) Pulmonary fibrosis
b) Pulmonary hypertension
c) COPD
d) Obesity

A

b) Pulmonary hypertension

Up to date: Overview of pulmonary function testing in adults

Diffusing capacity — Measurement of the single-breath diffusing capacity for carbon monoxide (DLCO; also known as transfer factor or TLCO) is quick, safe, and useful in the evaluation of restrictive and obstructive lung disease, as well as pulmonary vascular disease. The technique and interpretation are discussed separately.

In the setting of restrictive disease, the diffusing capacity helps distinguish between intrinsic lung disease, in which DLCO is usually reduced, and other causes of restriction, in which DLCO is usually normal.

In the setting of obstructive disease, the DLCO helps distinguish between emphysema, in which it is usually reduced, and other causes of chronic airway obstruction, like asthma or chronic bronchitis, where it is usually normal.

The DLCO is also used in the assessment of pulmonary vascular disease (eg, thromboembolic disease, pulmonary hypertension), which typically causes a reduction in DLCO in the absence of significant restriction or obstruction

104
Q

21.1, 21.2, 20.1 Hepcidin production is inhibited in response to

a. Anaemia
b. Inflammation
c. Acute leukemia
d. Infection
e. Excess iron stores

A

anaemia

Iron deficiency can be caused by depletion of total Iron stores or a chronic loss of blood.

Metabolism of Iron is also influenced by disease states including inflammation and malignancy.

Raised Iron stores and inflammation upregulate the production of HEPCIDIN, a hormone responsible for the inhibition of enteral Iron absorption

HEPCIDIN degrades iton trans-membrane transporter ferroportin on duodenal enterocyte membranes. it also inhibits the transport of stored iron from hepatocytes and macrophages into plasma in a similar manner.

Upregulation of HEPCIDIN can produce functional iron deficiency, lading to what has been tradionally known as the anaemia of chronic disease

HEPCIDIN deficiency is the cause of iron overload in hereditary hemochromatosis, iron-loading anemias, and hepatitis C

HEPCIDIN is suppressed in iron deficiency, allowing increased absorption of dietary iron and replenishment of iron stores. Increased erythropoietic activity also suppresses HEPCIDIN production.

HEPCIDIN is decreased in iron deficiency anemia, hemolytic anemia, and anemias with ineffective erythropoiesis

105
Q

22.2 A 21-year-old patient with a history of schizophrenia on quetiapine develops tremor, restlessness, hyperreflexia, nausea and vomiting in the post-anaesthesia care unit following an emergency laparoscopic cholecystectomy. Her heart rate is 80 / minute, blood pressure 130/90 mmHg, and her temperature is 37.0°C. The most likely diagnosis is

a. MH
b. NMS
c. serotonin syndrome
d. rhabdomyolysis
e. anticholinergic crisis

A

Serotonin Syndrome
Hyper reflexia
Usually has hypertension and hyperthermia

https://static1.squarespace.com/static/5e6d5df1ff954d5b7b139463/t/617242e2ab18df2dee31f417/1634878179720/ICU_one_pager_hyperthermic_toxidromes.png

106
Q

20.1 A condition that is NOT associated with a raised baseline serum mast cell tryptase level is

a) Chronic renal failure
b) Alcoholic liver disease
c) Chronic eosinophilic leukaemia
d) Mastocystosis
e) Acute myeloid leukaemia

A

b) Alcoholic liver disease

A raised serum tryptase (hypertryptasemia) has numerous diagnostic uses. The main use is in the diagnosis of anaphylactic reactions; however, it does not distinguish between IgE-mediated versus non-IgE-mediated reactions.

Tryptase is a serine protease primarily produced by MCs, and to a lesser extent by mature basophils and myeloid progenitors. (Eosinophils have myeloid progenitor)

Elevated serum tryptase is an independent prognostic factor for progression to ESRF in patients treated with ACEi or ARB

https://associationofanaesthetists-publications.onlinelibrary.wiley.com/doi/10.1111/j.1365-2044.2004.03757.x

Systemic Anaphylaxis:
levels peak at 15-120mins and with half life of 1.5-2.5hrs

histamine levels peak at 5 mins and decrease to baseline within 15-30mins

B-Tryptase testing can be performed on blood samples obtained 1-6hrs after onset

Hihg tryptase concentration can be found 3 days after death from suspected anaphylaxis

Other allergic conditions:
Asthma
In asthma, β-tryptase is usually overexpressed or released from mast cells prematurely. It causes a cascade of events such as airway inflammation and bronchoconstriction. A mutation of the β-tryptase gene causes the overexpression

Sudden Infant Death Syndrome:
Anaphylaxis has been suggested to be a cause of the sudden infant death syndrome (SIDS), although allergen sensitivity and mast cell activation have not been demonstrated. Elevated postmortem levels of β-tryptase in victims of SIDS have been reported.

Amniotic Fluid Embolism:
A case study of an autopsy-proven fatal case of amniotic fluid embolism reported a significantly elevated serum β-tryptase level, suggesting possible mast cell activation. However, there are other reported cases of amniotic fluid embolism diagnosed clinically whose tryptase levels were not elevated.

Systemic Mastocytosis:
Characterised by mast cell hyperplasia in bone marrow, skin, liver, spleen and gastrointestinal mucosa.
A study of tryptase levels in patients with biopsy-proven mastocytosis reported concentrations of total tryptase > 20 ng.ml−1 and ratios of total tryptase to β-tryptase > 20, whereas normal patients had total tryptase levels < 14 ng.ml−1

MC activation Syndrome
These heterogeneous conditions are often associated with biochemical evidence of MC activation, but total tryptase may be normal.

Urticaria/Agioedema:
Basal total serum tryptase tends to be higher in patients with chronic urticaria over atopic and non-atopic healthy controls, likely due to increased MC burden and/or greater release of tryptase per MC. Other studies have found tryptase ≥15 μg/L in 12% of patients with urticaria/angioedema without anaphylaxis

Familial Hypertryptasaemia
These patients have a baseline tryp­tase of >8 μg/L and commonly have complaints of arthralgias, gastroesophageal reflux disease, flushing, urticaria, hypermobility, postural orthostatic tachycardia syndrome, increased risk of anaphylaxis to stinging insects, and irritable bowel syndrome

CKD and ESRF
Tryptase may be raised in patients with chronic kidney disease (CKD), accounting for around 7% of all elevated tryptase samples in one laboratory’s retrospective review of elevated tryptases. It tends to be elevated with more severe CKD, correlating with other markers of CKD including creatinine and proteinuria

Haematological conditions:
Hypereosinophilic syndrome (HES) is a heterogeneous group of systemic diseases of unknown cause characterised by excessive eosinophils invading the heart, lungs, brain and nerves, causing organ damage
Patients with HES and elevated serum tryptase levels were more likely to develop fibroproliferative end organ damage and had a shorter life expectancy.

Myelodysplastic syndrome (MDS) is a large group of acquired neoplastic disorders of the bone marrow most common in the elderly and is caused by an abnormal differentiation and maturation of haemopoietic cells.
Elevated levels of total tryptase (α-tryptase and β-tryptase) are found in a group of patients with MDS

Parasitic infection
MCs and tryptase play a significant role in parasitic infections [72], so it is of no surprise that serum tryptase can be elevated in these infections.

Gausher’s disease
Rare genetic disorder caused by the deficiency of β-glucocerebrosidase, an enzyme for the catabolism of glucocerebroside, a component of cell membranes.
A single case report exists of an 18-month-old boy who had an immediate hypersensitivity to imiglucerase enzyme replacement and an elevated baseline tryptase of 63.2 μg/L.

107
Q

20.1 Patients with obstructive sleep apnoea undergoing surgery, have been shown to have an increased incidence of

A) AF
B) Acute renal failure
C) AMI
D) Perioperative mortality
E) Unplanned admission after ambulatory surgery

A

C) AMI

The association between OSA and perioperative mortality is unclear [19], with various studies showing increased [12,23] or comparable [14] mortality. While decreased mortality [7,18,43] has been reported in retrospective studies, this may be due to unrecognized or undiagnosed OSA in the control or non-OSA groups, and/or the possibility that patients with diagnosed OSA received monitoring and treatment of their OSA.

SOBA 2015:
Severe OSA occurs in 10–20% of patients with BMI > 35 kg.m2 and is often undiagnosed. Overall, a diagnosis of OSA is associated with a greater than doubling of the incidence of postoperative desaturation, respiratory failure, postoperative cardiac events and ICU admission [16]. The presence of multiple and prolonged oxygen desaturations increases the sensitivity to opioid-induced respiratory depression [17]. However, if identified pre-operatively and treated appropriately with continuous positive airway pressure (CPAP), the risk of complications is much reduced [18].

Obesity has no influence on the rate of unanticipated admission, postoperative complications, readmission or other unplanned contact with health professionals after home discharge.

108
Q

21.1 A patient with a history of hereditary angioedema requires an appendectomy for acute appendicitis.
The most effective therapy for the prevention of an acute attack in the perioperative period is

a) FFP
b) Icatibant
c) Hydrocortisone
d) Danazole
e) cetirizine

A

d) Danazol
https://www.allergy.org.au/hp/papers/hereditary-angioedema

Treatment options:
Plasma derived C1-esterase inhibitor = Berinert/Cinryze,
Androgens = Danazol
B2 Bradykinin REceptor antagonist = Icatibant
FFP.

Danazol (an androgen) is recommended as first line PROPHYLAXIS for planned procedures (need to give for 5-10 days prior and 2-5 days post)

For emergency or high risk procedures C1 esterase inhibitor concentrate (Berinert or Cinryze) is recommended
- give 1 hour before procedure
- more effective than danazol but more expensive

Berinert:
- 20units/kg IV over 10 min
- Symptoms usually stabilise in 30 mins
- 2nd dose uncommon, but may be given 30mins to 2hrs after 1st dose

Icatibant:
- 30mg slow subcut infusion in abdominal area

Due to the risk of precipitating laryngeal oedema, oropharyngeal procedures should usually involve general anaesthesia with endotracheal intubation

Short answer:
- if you have days before surgery increase danazole, if complex surgery increase danazole and give C1Inh
- If you have acute emergency surgery give C1Inh Concentrate (Berinert/Cinryze) before and after
- if you have an acute attack use C1Inh or Bradykinin antagonist (Icatibant)
- If C1 Inh and Bradykinin antagonoist are not available then use FFP but this may worsen the attack due to the presence of C4 in the FFP
- Has Cetirizine been misremembered instead of Cinryze as an option in this question? No it wasn’t
-> adrenaline, steroids, antihistamines have no role in treatment of HAE acute attack

109
Q

22.2 The electrolyte abnormality most associated with an increased risk of laryngospasm is
a. Hypokalaemia
b. Hyponatraemia
c. Hypocalcaemia
d. Hypercalcaemia
e. Hypernatraemia

A

c. Hypocalcaemia

Laryngospasm is a rare, but serious and potentially lethal, complication of hypocalcemia in adults. In every adult presenting with acute dyspnea and stridor, the possibility of hypocalcemia should be considered. Hypocalcemia should be treated promptly.

110
Q

21.1 All of the following conditions are associated with acromegaly EXCEPT

a. Myocardial fibrosis
b. biventricular enlargement
c. Arrhythmia
d. Left ventricular enlargement
e. AAA

A

e. AAA

Osteoarthritis
nerve compression syndrome due to bony overgrowth, and carpal tunnel syndrome
Hypertension
Diabetes mellitus
Cardiomyopathy/HF
Colorectal cancer
Sleep Apnea
Thyroid nodules and thyroid cancer
Hypogonadism
Compression of the optic chiasm

Source: BJA

111
Q

22.1 Of the following clinical conditions, difficult intubation is LEAST likely to be associated with

a. Apert syndrome
b. Hurler
c. Pierre Robin
d. Downs Syndrome
e. Treacher collins

A

d. Downs Syndrome

112
Q

The risk of developing postherpetic neuralgia may be reduced by treating acute herpes zoster (shingles) with

A. Ibuprofen
B. Gabapentin
C. Aciclovir
D. Amitriptyline
E. Oxycodone

A

D. Amitriptyline

Amitriptyline (used in low doses for 90 days from onset of the herpes zoster rash) reduces the incidence of postherpetic neuralgia

N.B
Antiviral agents started within 72 hours of onset of the herpes zoster rash accelerate the resolution of acute pain (U) (Level I) but do not reduce the incidence, severity and duration of postherpetic neuralgia

UTD
Both Gabapentinoids and TCAs are effective at TREATING postherpetic neuralgia. The former have lower risk of discontinuation due to adverse side effects.
For moderate or severe pain, use gabapentinoids.

113
Q

A patient’s glomerular filtration rate is estimated at 35 mL/min/1.73m2. The patient’s chronic kidney disease can be classified as Stage
a) 2
b) 3a
c) 3b
d) 4
e) 5

A

c) 3b

114
Q

Diffusing capacity of the lungs for carbon monoxide (DLCO) is decreased in all of the following EXCEPT

made up potential answers:

a) Pulmonary Fibrosis
b) Interstitial Lung disease
c) Obesity
d) Pulmonary haemorrhage

A

d) Pulmonary haemorrhage

Rewording of 21.2 Question

Won’t increase in Myasthenia Gravis

Causes of HIGH value include:
Asthma
Left-right intracardiac shunt
polycythaemia
Pulmonary haemorrhage
Obesity - Dlco will increase but kco will not

115
Q

The condition for which you would have a lower arterial oxygen saturation target is

Made up responses

a) Carbon monoxide poisoning
b) Bronchopulmonary Pneumonia
c) Bleomycin toxicity
d) Pulmonary Fibrosis

A

Answers could also possibly be COPD, Acute stroke or Neonates

c) Bleomycin toxicity

Bleomycin is a particularly important chemotherapy drug for the anaesthetist to be aware of. Bleomycin is often used to treat germ cell tumours and Hodgkin’s disease in a curative setting. The major limitation of bleomycin therapy is the potential for subacute pulmonary damage that can progress to life-threatening pulmonary fibrosis. Pulmonary toxicity occurs in 6–10% patients and can be fatal.2 Exposure to high-inspired concentration oxygen therapy, even for short periods, as experienced during anaesthesia, is often implicated in causing rapidly progressive pulmonary toxicity in patients previously treated with bleomycin.3 These claims have been considered controversial by some, but it is the authors’ recommendation that any patient previously exposed to bleomycin therapy should be treated as high risk, and summary guidance regarding oxygen therapy is shown in Table 4.

Summary guidance—oxygen therapy for patients who have received bleomycin > Patients have a life-long risk of bleomycin-induced lung injury
> Oxygen therapy should be avoided if at all possible
> Clinical procedures (and leisure activities) involving a high should be avoided If a patient is hypoxic
> O2 therapy should be minimized to maintain O2 saturation of 88–92%
> High oxygen concentrations should be used with extreme caution for immediate life-saving indications only (to maintain O2 saturation of 88–92%)

116
Q

A 30 year old parturient presents in labour. She has a history of Addison’s disease from
autoimmune adrenalitis and has been taking prednisolone 6 mg daily for ten years. On
presentation the patient is given hydrocortisone 100 mg intravenously. The most appropriate steroid replacement regime the patient should receive during labour is

a. 25mg TDS hydrocortisone
b. 8mg/hr hydrocortisone
c. 6mg PO prednisone

A

8mg/hr

Guidelines for mx of glucocorticoids during the perioperative period for patients with adrenal insufficiency

https://associationofanaesthetists-publications.onlinelibrary.wiley.com/doi/10.1111/anae.14963

117
Q

Hepatopulmonary syndrome can be treated with

a) Methylene blue
b) Inhaled nitric oxide
c) Nitric oxide inhibitors
d) Oxygen therapy
e) Liver transplantation

A

e) Liver transplantation

  • Oxygen therapy for symptom relief
  • Liver transplant provides long term survival benefit
  • All other therapies tried but no conclusive evidence of benefit/nil are FDA approved

Hepatopulmonary Syndrome Article https://www.ncbi.nlm.nih.gov/books/NBK562169/

Hepatopulmonary syndrome (BJA)
- Prevalence up to 20% (end stage liver disease)
- Characterised by: disordered pulmonary capillary vasodilation and VQ mismatch
- Present with hypoxia, ortheodeoxia (decrease in PaO2 when standing)
- Diagnosis w/bubble echocardiography
- Risk factor for early post-transplant mortality
- If transplant successful, will resolve over time

118
Q

Individuals with Prader-Willi syndrome having an anaesthetic are at most risk of

a) Hypocalcaemia
b) Hypoglycaemia
c) Neuroleptic malignant syndrome
d) Malignant hyperthermia
e) Hypothermia

A

b) Hypoglycaemia

Stoelting:
Prader-Willi syndrome is a rare genetic disorder characterized by
hypothalamic-pituitary abnormalities with severe hypotonia during the neonatal period and
during the first two years of life, hyperphagia with a risk of morbid obesity during infancy and
adulthood, learning difficulties and behavioural problems or severe psychiatric problems. The
disease affects 1/25,000 births.

https://www.orphananesthesia.eu/en/rare-diseases/published-guidelines/prader-willi-syndrome/1339-prader-willi-syndrome-2/file.html

Prader-Willi syndrome is a rare genetic disorder characterized by
hypothalamic-pituitary abnormalities with severe hypotonia during the neonatal period and
during the first two years of life, hyperphagia with a risk of morbid obesity during infancy and
adulthood, learning difficulties and behavioural problems or severe psychiatric problems. The
disease affects 1/25,000 births.

119
Q

Refeeding syndrome following the commencement of total parenteral nutrition is associated with the development of

A

Most likely answer will be related to hypophosphataemia

Refeeding syndrome is a constellation of biochemical abnormalities which occurs when normal intake is resumed after a period of starvation. Its characteristic features are low levels of phosphate, potassium, magnesium and sodium. Its major complications include cardiac arrhythmias, heart failure (due to hypophosphataemia), muscle weakness, rhabdomyolysis, seizures and an altered sensorium.

The major risk factors are calorie malnutrition of any cause, alcohol or drug use, low BMI (18-16) and starvation for 5-10 days.

Pathophysiology
With the restoration of glucose as a substrate, insulin levels rise and cause cellular uptake of these ions. Depletion of adenosine triphosphate (ATP) and 2,3-diphosphoglyceric acid (2,3-DPG) results in tissue hypoxia and failure of cellular energy metabolism. This may manifest as cardiac and respiratory failure, with paraesthesiae and seizures also reported. Thiamine deficiency may also play a part.

  • Exogenous sources of phosphate are inadequate to supplement the daily phosphate requirements
  • Intracellular phosphate stores are used to synthesise ATP (using protein and fat as fuel)
  • Homeostatic mechanisms maintain serum concentrations of these ions at the expense of intracellular stores

Reference: https://derangedphysiology.com/main/required-reading/endocrinology-metabolism-and-nutrition/Chapter%20315/refeeding-syndrome “

120
Q

The most appropriate initial diagnostic test for a suspected phaeochromocytoma is a/an

A

serum free metanephrines and nor-metanephrines

https://www.ncbi.nlm.nih.gov/pmc/articles/PMC3230088/

https://www1.racgp.org.au/ajgp/2021/january-february/adrenal-disease-an-update

121
Q

REview Duchenne muscular dystrophy is NOT associated with:

a) Increased CK
b) Cardiomyopathy in female carriers
c) decreased Sensitivity to non-depolarising NMBs

Alternative remembered answers:

a) Reisistant to NDNMB
b) Premature death
c) Aspiration
d) Conduction abnomality in females

A

Increased sensitivity to non depolarisers

Ck -> Anaesthesia induced rhabdo
Cardio- All at-risk females, regardless of their carrier status, should be monitored for development of cardiomyopathy

122
Q

NP: The antibiotic considered safest to be administered to a patient with myasthenia gravis in the perioperative period is:

a) Vancomycin
b) Gentamycin
c) Erythromycin
d) Flucloxacillin
e) Ciprofloxacin

A

d) Flucloxacillin

Need remembered options:
Black box warning for fluoroquinolones (ciprofloxacin)

Probably also avoid
Aminoglycosides (Amikacins/gentamicin/streptomycin) and tobramycin although TOBRAMYCIN probably least problematic of these.
Macrolides (erythromycin)

These antibiotics have not been shown to cause many problems for MG patients
Tetracycline (doxycycline, minocycline) – this may worsen MG
Sulfonamides (Bactrim), Penicillin – causes rare cases, usually not a problem for majority of MG patients

https://myastheniagravis.org/mg-and-drug-interactions/#:~:text=These%20antibiotics%20have%20black%20box,Ketek%20(telithromycin)

123
Q

Steph In a patient presenting with an Addisonian crisis, the electrolyte disturbances MOST LIKELY to be seen are:

a) Low BSL, hyperkalaemia, hyponatraemia
b) High BSL, hyperkalaemia, hyponatraemia
c) Hypocalcaemia, hyperkalaemia, hyponatraemia
d) Hypercalcaemia, hyperkalaemia, hyponatraemia

A

a) Low BSL, hyperkaelamia, hypernatraemia

Adrenal crisis is a medical emergency and should be considered in any patient presenting with one or more of the following symptoms:
* altered consciousness
* circulatory collapse
* hypoglycaemia
* hyponatraemia
* hyperkalaemia
* seizures
* history of steroid use/withdrawal
* any clinical features of Addison disease

Adrenal crisis may be precipitated by stress, sepsis, dehydration or trauma; clinical features may be modified accordingly. In patients with known adrenal insufficiency, nonadherence with therapy, inappropriate cortisol dose reduction or lack of stress related cortisol dose adjustment can cause adrenal crisis.

Aus Family Physician - RACGP

Re chat below - incorrect recall, have updated
A

Why A? All three should be seen - glucocorticoid deficiency causes low Na and glucose while simultaneous mineralocorticoid deficiency low K.

Crisis typically presents with hypotension abdo pain, nausea, vomiting and confusion. No one electrolyte/lab value can tie all those together.

124
Q

23.1 A feature that is atypical of multiple sclerosis is

A. Unilateral visual loss
B. Aphasia
C. Diplopia
D. Lower limb motor
E. Some sensory thing

A

B. Aphasia

UTD

125
Q

20.1 Perioperative overheating is most likely to cause worsening of symptoms of

A) Duchenne Muscular dystrophy
B) Myasthenia gravis
C) Multiple sclerosis
D) Myotonica dystrophia
E) Eaton Lambert syndrome

A

Answer: c) MS

CEACCP 2012 Neuromuscular disorders and anaesthesia. Part 2: specific neuromuscular disorders

Multiple sclerosis
This is the most frequently occurring demyelinating neuromuscular disorder. It is a chronic relapsing condition characterized by the formation of plaques within the brain and spinal cord. These plaques cause demyelination around the axons, resulting in weakness and spasticity as well as sensory dysfunction.
Anaesthetic considerations. Local anaesthetics may exacerbate symptoms due to the increased sensitivity of demyelinated axons to local anaesthetic toxicity.
Non-depolarizing neuromuscular blocking agents may be used in normal doses. Caution should be exercised when using depolar- izing neuromuscular blocking agents if the patient is debilitated. Temperature maintenance is important as symptoms can deteriorate with an increase in temperature, as demyelinated axons are also more sensitive to heat.

BJA: Perioperative management of myasthenia gravis (2021 - written after this MCQ):

Several factors, many associated with surgery and anaesthesia, may exacerbate myasthenia or lead to a myasthenic crisis, a life-threatening condition in which severe respiratory muscle insufficiency leads to respiratory failure.
Crises are most commonly precipitated by infection. Other precipitants include surgery, residual neuromuscular block, pain, many drugs, hypo- and hyperthermia, reduction or withdrawal of treatment, pregnancy, stress and sleep deprivation.